SlideShare una empresa de Scribd logo
1 de 174
STROKE &
CEREBROVASCULAR
ACCIDENT
MOHD HANAFI RAMLEE
Definition
• Transient Ischaemic Attack (TIA) – an acute focal
neurological deficit resulting from cerebrovascular
disease with resolution of signs and symptoms
within 24 hours.
• Reversible Ischaemic Neurological Deficit (RIND) –
attack lasting longer than 24 hours but with
complete clearance of signs and symptoms within 7
days.
• Completed Stroke – neurological deficit lasts longer
than 7 days.
HOW YOU GIVE THE
DIAGNOSIS?
• The diagnosis should provide answers to the following
questions:
• 1. What is the neurological deficit?
• 2. Where is the lesion?
• 3. What is the lesion?
• 4. Why has the lesion occurred?
• 5. What are the potential complications and prognosis?
What Artery Involved?
• Aphasia (dominant hemisphere)
• Hemiparesis / plegia
• Hemisensory loss/disturbance
• Homonymous hemianopia
• Parietal lobe dysfunction, e.g. astereognosis, agraphaesthesia,
impaired two-point discrimination, sensory and visual
inattention, left-right dissociation and acalculia
What Artery Involved?
• Weakness of lower limb more than upper limb
What Artery Involved?
• Homonymous hemianopia
• Cortical blindness
• Ataxia
• Dizziness or vertigo
• Dysarthria
• Diplopia
• Dysphagia
• Horner’s syndrome
• Hemiparesis or hemisensory loss contralateral to the cranial
nerves palsy
• Cerebellar signs
SO… BASIC IS
IMPORTANT!!!!!!!!
Brain Blood Supply Features:
• High oxygen requirement.
• Brain 2% of body weight - 15% of cardiac output
• 20% of total body oxygen.
• Continuous oxygen requirement
• Few minutes of ischemia - irreversible injury.
• Neurons - Predominantly aerobic.
• Sensitive areas:
• Adults -Hippocampus,
3,5th & 6th layer of cortex,
Purkinje cells.
Border zone (watershed areas)
• Brain stem nuclei in infants.
Anatomy – Stroke.
Brodman’s Cortical Map:
Frontal[f*ck–motor]Lobe Functions:
• High level cognitive functions. i.e reasoning, abstraction,
concentration
• Storage of information – memory
• Control of voluntary eye movement
• Motor control of speech in the dominant hemisphere.
• Motor Cortex – Motor control of the contralateral side of
the body
• Urinary continence
• Emotion and personality
Parietal[p-sx–sensory]Lobe Functions:
• Sensory cortex – sensory input is interpreted to define
size, weight, texture and consistency (contralateral)
• Sensation is localised, and modalities of touch, pressure
and position are identified.
• Awareness of the parts of body
• Non-dominant – processes visuospatial information and
• controls spatial orientation
• Dominant is involved in ideomotor praxis (ability to
perform learned motor tasks
Temporal[t-telinga]Lobe Functions:
• Primary auditory receptive areas
• In dominant ability to comprehend speech
(wernicke’s) – reception
• Interpretive area – area at the junction of the
temporal, parietal and occipital lobes.
• Plays an important role in visual, auditory and
olfactory perception
• Important role in learning; memory and
emotional affect.
Occipital[O-optic]Lobe Functions:
• Primary visual cortex
• Visual association areas
• Visual perception
• Some visual reflexes (i.e. visual
fixation)
• Involuntary smooth eye movement
Diencephalon Functions:
• Brain Stem:
• Midbrain, Pons & Medulla
• 10 of the 12 ranial nerves arise from the brainstem
(ipsilateral signs)
• Cortical pathway decussation contralateral signs.
• Some major functions: eye movement, swallowing,
breathing, blood pressure, heat beat, consciousness
• Cerebellum:
• movement – Balance & coordination
Motor & Sensory Cortex:
Diencephalon & Brain stem:
Cranial Nerves:
Stroke Types:
• Clinical
• Transient Ischemic Attack –TIA resolve <24h
• Evolving stroke – increasing >24h. – Thromb.
• Recurrent / multiple stroke – sec. factors.
• Completed stroke – no change… embolic.
• Pathological
• Focal / Global
• Ischemic & hemorrhagic (chronic/acute)
• Venous infarcts. (young, infections)
Common Types and Incidence:
• Infarction: Incidence 80% - mortality 20%
• 50% - Thrombotic – atherosclerosis
• Large-vessel 30% (carotid, middle cerebral)
• Small vessel 20% (lacunar stroke)
• 30% Embolic (heart dis / atherosclerosis)
• Young, rapid, extensive.
• Venous thromboembolism (rare)
• Hemorrhage: Incidence 20% - mortality 80%
• Berry aneurysm, Microaneurysm, Atheroma.
• Intracerebral or subarachnoid.
Stroke location and incidence:
Cause %
Clinical
presentation
30day
mort(%) Pathogenesis
Cerebral
infarction
85 Slowly / sudden
evolving signs and
symptoms
15-45 Cerebral
hypoperfusion
Embolism
Thrombosis
Intracerebral
hemaemorrha
ge
10 Sudden onset of
stroke with raised
intracranial
pressure
80 Rupture of micro-
aneurysm or arteriole
Subarachnoid
haemorrhage
5 Sudden headache
with meningism
45 Rupture of saccular
aneurysm on circle of
Willis
Clinical Categories:
• Global Ischemia.
• Hypoxemic encephalopathy
• Hypotension, hypoxemia, anemia.
• Focal Ischemia.
• Obstruction to blood supply to focal
area.
• Thrombosis, embolism or hemorrhage.
GLOBAL ISCHAEMIA
Global Ischemia:
• Etiology:
• Impaired blood supply - Lung & Heart disorders.
• Impaired O2 carrying – Anemia/Blood dis.
• Impaired O2 utilization – Cyanide poisoning.
• Morphology:
• 3rd, 5th and 6th layers of the cortex, hippocampus and in the
Purkinje cells in the cerebellum
• Laminar necrosis, Hippocampus, Purkinje cells.
• Border zone infarcts – “Watershed”
• Sickle shaped band of necrosis on cortex.
• Clinical Features:
• Mild transient confusion state to
• Severe irreversible brain death. Flat EEG, Vegetative state. Coma.
Morphology in Global Ischemia
1. Watershed zone
(Acute - ACA-MCA)
2. Laminar necrosis -
(chronic- short penetrating
arteries)
3. Sommer sector of
hippocampus.
4. Purkinje cells of
cerebellum.
Watershed/Boundary zone infarcts:
Carotid thrombosis
Lamellar necrosis in global ischemia.
Chronic
Local infarction:
Cell death ~
6min
central infarct
area or umbra,
surrounded by
a penumbra of
ischemic tissue
that may
recover
Infarct Pathogenesis:
• Reduced blood supply – hypoxia/anoxia.
• Altered metabolism  Na/K pump block.
• Glutamate receptor act.  calcium influx.
• ischemic injury – Red neuron, vacuolation.
• cell death, karyorrhexis.
• Inflammation – edema.
• Macrophages - > 5d.
• Liquifaction cavity – >1wk
• Glial proliferation – >1wk. (astrocytes)
Hours
1-day
3-day
1 wk.
>4wk
Infarct Stages:
• Immediate – <24 hours
• No Change gross, micro  Na/K loss, Ca+ influx.
• Acute stage – < 1week
• Oedema, loss of grey/white matter border.
• Inflammation, Red neurons, necrosis, neutrophils
• Intermediate stage – 1- 4 weeks.
• Clear demarcation, soft friable tissue, cysts
• Macrophages, liquifactive necrosis
• Late stage – > 4 weeks.
• Removal of tissue by macrophages
• Fluid filled cysts with dark grey margin (gliosis)
• Gliosis – proliferation of glia at periphery.
FOCAL ISCHAEMIA
Brain Stem Stroke:CommonPattern
• Pure Motor - Weakness of face and limbs on one side of the
body without abnormalities of higher brain function,
sensation, or vision (MCA/ACA)
• Pure Sensory - Decreased sensation of face and limbs on one
side of the body without abnormalities of higher brain
function, motor function, or vision (PCA).
MCA [mostcommon]Features:
• Paralysis of the contralateral
face, arm and leg
• Sensory impairment over the
contralateral face, arm & leg
• Homonymous hemi or
quadrantonopia
• Paralysis of gaze to the
opposite side
• Aphasia (dominant) and
dysarthria [broca/wernicke]
• Penetrating - contralateral
hemiplegia/paresis, slurred
speech.
• Impaired spatial perception
MCA stroke.
MCA stroke.
Wikipedia: GNU Free Documentation license
MCA stroke.
Wikipedia: GNU Free Documentation license
ACA stroke.
• Paralysis of contralateral foot and
leg
• Sensory loss over toes, foot and
leg
• Impairment of gait and stance
• Abulia (slowness and prolonged
delays to perform acts)
• Flat affect, lack of spontaneity,
slowness, distractibility
• Cognitive impairment, such as
perseveration and amnesia
• Urinary incontinence
Wikipedia: GNU Free Documentation license
PCA stroke.
Peripheral (cortical)
• Homonymous hemianopia
• Memory deficits
• Perseveration
• Several visual deficits (cortical blindness,
lack of depth perception, hallucinations)
Central (penetrating)
• Thalamus - contralateral sensory loss,
spontaneous pain, mild hemi
• Cerebral peduncle - CN III palsy with
contralateral hemiplegia
• Brain stem - CN palsies, nystagmus,
pupillary abnormalities
Wikipedia: GNU Free Documentation license
Posterior Cerebral Artery
• Visual disturbances
• contralateral homonymous
hemianopsia
• (central vision is often spared)
• L. Hemi: lesions alexia
• (with or without agraphia)
• Bilateral lesions: cortical
blindness
• patients unaware they cannot see
• (Anton's syndrome)
• Memory impairment if temporal
lobe is affected ~
• Proximal occlusion
• contralateral hemisensory
loss,
• spontaneous pain and
dysesthesia if thalamus
affected
• (thalamic pain syndrome)
• contralateral severe proximal
chorea
• (hemiballism) ~
Haemorrhagic - Arterial embolus
Embolic stroke: sudden, pin point hemorrhages over a triangular area.
Infarct with Punctate hemorrhage
Cerebral Infarction - Late
Cerebral Infarction - Late
Hypertensive CVD
• Intraerebral/Subarachnoid Hemorrhage
• Microaneurysm hemorrhages – Basal ganglia.
Putamen(60%), thalamus, ventricles.
• Berry aneurysm hemorrhages – subarachnoid.
• Chronic Hypertension: (dementia)
• Slit hemorrhages. Microhemorrhages heal as slit with
pigment.
• Lacunar infarcts: Brain stem - pale infarcts. A.sclerosis
• Hypertensive encephalopathy-Malignant.
• Headache, confusion, vomiting – Raised ICP.
Central PontineHemorrhage- Herniation
Subarachnoid Hemorrhage:
Ruptured Berry Aneurism
Fusiform
atherosclerotic
aneurysm
Saccular(berry) Aneurysm:
Intracerebral Hemorrhage:
Lacunar Infarct in pons
Left(Dominant)HemisphereStroke:Clinical
• Aphasia
• Right hemiparesis
• Right-sided sensory loss
• Right visual field defect
• Poor right conjugate gaze
• Dysarthria
• Difficulty reading, writing, or
calculating
Diagnosis: Recent cerebral infarction in left MCA distribution.
Left cerebral hemisphere shows swelling with compression of the lateral ventricle
mainly in the frontal area, due to recent infarct in the Middle Cerebral Artery (MCA)
distribution. The brain in the MCA area shows discoloration of the cortex and also
blurring between the cortex and white matter.
Right(Non-dominant)-HemisphereStroke:
• Defect of left visual field
• Extinction of left-sided
stimuli
• Left hemiparesis
• Left-sided sensory loss
• Left visual field defect
• Poor left conjugate gaze
• Dysarthria
• Spatial disorientation
CNS AV Malformations:
• Many types:
• AV Malformation *
• Cavernous angioma
• Telangiectasia
• Venous angioma
• Cause of Seizure disorders
& hemorrhage.
• Most common congenital
vascular malformation.
• Typically located in the
outer cerebral cortex
underlying white matter.
Pathological Review:
• Stroke: Ischemic / Thrombotic / Hemorrhagic
• Acute neurological deficit - Clinical
• Cerebro Vascular Accident – Pathology.
• Etiology: Thrombosis, Embolism, Hemorrhage.
• Risk factors: AS, Hypertension, Smoking.
• Global – Systemic Hypoxia – Watershed & lamellar infarct
• Focal – Basal ganglia, Putamen, Int. capsule (MCA)
• Pathogenesis: Infarction  Liquifaction necrosis  Cyst formation
with peripheral gliosis. (loss of neural function)
• Hypertension & CVA:
• Atherosclerosis - Thrombosis
• Haemorrhage (Intra/subarachnoid),
• chronic benign: Lacunar infarcts & slit hemorrhages.
• Hypertensive Encephalopathy,
Stroke – Risk Factors
• Modifiable
• Hypertension
• Tobacco use
• Excess Alcohol
• Hx of TIA’s
• Heart Disease
• Diabetes Mellitus
• Hypercoagulopathy
• Pregnancy, cancer, etc.
• Sickle Cell and
increased RBC
• Hx of carotid Bruit
• Unmodifiable
• Age
• Gender
• Race
• Previous CVA
• Heredity
Stroke – Signs and Symptoms
• Ischemic
• Carotid Circulation
• Unilateral paralysis (opposite side)
• Numbness (opposite side)
• Language disturbance
• Aphasia – difficult comprehension, nonsense, difficult reading/writing
• Dysarthria – slurred speech, abnormal pronunciation.
• Visual disturbance (opposite side)
• Monocular blindness (same side)
Stroke – Signs and Symptoms
• Ischemic
• Vertebrobasilar Circulation
• Vertigo
• Visual disturbance
• Both eyes simultaneously
• Diplopia
• Ocular palsy – inability to move to one side
• Dysconjugate gaze – asynchronous movement
• Paralysis
• Numbness
• Dysarthria
• Ataxia
Stroke – Signs and Symptoms
• Hemorrhagic
• Subarachnoid hemorrhage
• Sudden severe HA
• Transient LOC
• Nausea/Vomiting
• Neck pain
• Intolerance of noise/light
• AMS
• Intracerebral hemorrhage
• Focal sx w/ LOC, N/V
History
• Detailed history from relative or friend or patient if he is able
to speak.
• Rapidity of onset – sudden onset of a focal neurological
deficit.
• Time course of symptoms – maximum deficit over seconds or
minutes before starting to improve.
• Headache, coma at onset and vomiting at onset are more
common in haemorrhage but also occur with infarction.
• Sudden onset of severe generalised headache associated with
neck stiffness – subarachnoid haemorrhage.
• Specific record should be made about the presence of vascular
risk factors.
Examination
General Examination
• BP – should be taken in both arms.
• Stroke may cause an acute rise in BP and therefore
hypertension should not be diagnosed in the first few days
after a stroke unless left ventricular hypertrophy of fundal
changes are present.
• Pulse – for arrhythmias particularly atrial fibrillation.
• Peripheral pulses.
• Auscultation for a carotid bruit.
• Heart – for valvular heart disease especially mitral stenosis.
• Neck stiffness – subarachnoid haemorrhage or meningitis.
• Identify the anatomical localization of the lesion and record
the degree of disability.
OXFORDSHIRE COMMUNITY STROKE
PROJECT CLASSIFICATION
OCSP
Causes of ischaemic stroke
Causes of ischaemic stroke 2
Causes of ischaemic stroke 3
Causes of intracranial
haemorrhage
Subarachnoid haemorrhage
• Initial headache or coma – sudden rise in intracranial pressure.
• Focal symptoms if aneurysm ruptures into underlying brain.
• Cerebral vasospasm causes delayed cerebral infarction 4-14
days after onset in 30% of patients.
• Recurrent haemorrhage and hydrocephalus are other
complications.
Intracerebral Haemorrhage
• Sudden rupture of microaneurysms caused by hypertensive
vascular disease.
• Characteristically occurs in the basal ganglia, pons and
cerebellum.
• Elderly patients – cerebral amyloid angiopathy, a degenerative
disorder affecting the walls of the artey – subcortical
haematomas.
• Cryptic av malformations are suspect especially in younger
patients < 40 yrs and when the haemotoma is Lobar (frontal,
temporal, parieto-occipital).
Investigations
Confirm the diagnosis
CT Scan
 To establish the site, size and pathological diagnosis by
showing infarction or haemorrhage.
 To exclude other conditions that may mimic stroke like
subdural haematoma, intracranial tumour.
(a) There is doubt about the diagnosis.
(b) Symptoms progress or fluctuate.
(c) Conscious level is depressed or patient is in coma.
(d) If thrombolytic therapy or anticoagulant treatment is
considered.
(e) Neck stiffness is present.
(f) Has severe headache.
(g) Deteriorates unexpectedly.
(h) Haemorrhage can be seen within a few minutes as an area of
increased attenuation.
(i) Infarction as a low density lesion which conforms to a
vascular territory usually wedge shaped.
(j) It is not immediately visible on CT but in most patients
becomes apparent in 4-7 days.
Y CT Scan?
MRI Scan
• Posterior circulation strokes are more readily identified than
by CT.
General Investigations
• identify conditions which may predispose towards premature
cerebrovasculardisease.
• Full blood count – polycythemia, thrombocytopoenia.
• Blood glucose – diabetes mellitus.
• Serum lipids – hypercholesterolemia.
• Blood cultures – SBE.
General Investigations II
• HIV screen – AIDS.
• Syphilis serology – VDRL.
• Clotting Screen.
• Thrombophilia Screen – Protein C, Protein S, Antithrombin III.
• Anticardolipin antibodies – SLE.
• Lumbar Puncture – subarachnoid haemorrhage.
DD Stroke
• Differential Diagnosis of Stroke
• Head/Cervical trauma
• Meningitis/encephalitis
• Hypertensive encephalopathy
• Intracranial mass
• Tumor
• Sub/epi dural hematoma
• Todd’s paralysis
• Migraine w/ neuro sx
• Metabolic
• Hyper/hypo glycemia
• Post arrest ischemia
• Drug OD
Differential Diagnosis
• Space occupying lesion
• 5% of people with stroke like symptoms have a subdural
haematoma, tumour or cerebral abscess.
• Distinction is readily made on CT or MRI.
• If there is any doubt repeat the scan after 6 weeks.
Differential Diagnosis II
• Multiple sclerosis
• May present with hemiparesis, sensory impairment or
brainstem symptoms that mimic stroke.
• Symptoms occur gradually over a few days.
• Hypoglycaemia
• May cause hemiparesis.
• Migraine
Complications I
• Cerebral oedema
•Should be suspected in a patient
with a large infarct or haemorrhage
experiences a lucid interval of 24-48
hours and then shows a decline in
consciousness.
Complication II
• Haemorrhagic transformation
• Can occur as a result of thrombolysis.
• Pneumonia
• In patients with swallowing difficulties as a result of aspiration.
• Pressure sores
• Develop rapidly and may be exacerbated because of
incontinence.
• Oedema of Weak Limbs
• Is common and has a partially autonomic basis.
Complication III
• Anxiety and Depression
• Common reactions to stroke but
depression may have an organic basis
related to damage of the frontal and
limbic systems.
• Emotional Lability
• Precipitated by minor emotional stimuli.
Management I
• Has the following aims
• Confirmation of the diagnosis,
anatomical site of the lesion, pathology
and aetiology.
• Acute care.
• Rehabilitation of persistent disability
and handicap.
• Prevention of recurrence.
Management II
Acute Care
Treatment aims
• Prevent progression of present event.
• Prevent immediate complication.
• Prevent the development of subsequent events.
• To rehabilitate the patient.
Management III
General Measures
• Around the edge of the infarct, ischaemic tissue is at risk, but is
potentially recoverable.
• This must be protected by ensuring a good supply of glucose
and oxygen.
• Maintain hydration, oxygenation and blood pressure.
Specific Measures
MedicalTreatment
Anticoagulation
• Patient with high risk of developing deep vein
thrombosis.
• Thromboembolic stroke - started as soon as
possible, except in large infarcts where it may
be wise to delay anticoagulation for 2 weeks.
• Stroke in a patient with myocardial infarct -
due to mural thrombus.
• Stroke in evolution.
Exclude a haemorrhage by doing a CT scan first.
Medical Treatment II
Antiplatelet Agents
• Especially in TIA.
Thrombolysis
• I/V thrombolysis espcially recombinant tissue plasminogen
activator rTPA to be given only within the first 3 hours after
onset to those patients who have not developed CT
abnormalities especially in patients with basilar artery
occlusion.
• Risk – haemorrhage.
Hypertension
• Treated cautiously in acute stroke.
• A reduction in blood pressure may lower cerebral blood flow in
the regions surrounding the infarct below a critical level at
which further ischaemic brain damage will occur.
• Mild to moderate elevations in BP – no treatment unless they
are maintained for several days after admission.
• If stroke associated with hypertensive encephalopathy or if
diastolic BP is persistently above 120 mm Hg.
• The BP should be lowered cautiously using oral agents.
• Sudden precipitious fall in BP should be avoided.
Hypertension II
Medical Treatment
CalciumAntagonist
• Nimodipine prevents ischaemic brain damage and reduces
the number of patients remaining disabled after
subarachnoid haemorrhage.
• Prescribed as soon as diagnosis is made (within 12 hours).
Neurosurgery
• Should be considered in subaracnoid and intracerebral
haemorrhage.
• Evacuation of cerebellar haematoma.
• Evacuation of supratentorial haematomas should only be
considered in younger patients with superficial cortical
haematomas causing mass effect with a deteriorating level of
conciousness.
Nursing Care and
Rehabilitation
• Physiotherapy, proper positioning and early mobilization –
prevent pressure sores.
• Support stockings – prevent deep vein thrombosis and
pulmonary embolism.
• Swallowing difficulties.
• Lead to silent aspiration – aspiration pneuomonia.
•Nasogastric tube feeding.
• Percutanious endoscopic gastrostomy.
Rehabilitation
• Physiotherapy, occupational therapy, speech therapy and
psychology input – multidisciplinary stroke rehabilitation
team.
• Home visit by occupational therapist to plan adaptations to
home before discharge.
Secondary Preventation
• Control hypertension and diabetes mellitus.
• Correct lipid abnormality.
• Stop cigarette smoking.
• Stop thrombogenic drugs e.g. oral contraceptives.
• Give platlet antiaggregation drugs to reduce the rate of
reinfarction.
• Low dose aspirin (75 mg – 150 mg), if
patient allergic or has gastrointestinal
side effects give ticlopidine. Regular
blood tests because of a small risk of
neutropoenia.
• Remove or treat embolic source (long term anticoagulation in
atrial fibrillation).
• Treat inflammatory or vascular inflammatory diseases.
Secondary Preventation II
• Carotid Endarterectomy – preventing stroke in symptomatic
patients with recent TIA and stroke and severe stenosis of the
internal carotid artery (at least 70%).
Secondary Preventation III
Chain of Survival Stroke
• Stroke Chain of Survival
• Detection
• Early sx recognition
• Dispatch
• Prompt EMS response
• Delivery
• Transport, approp, prehospital care, prearrival notification
• Door
• ER Triage
• Data
• ER evaluation incl, CT, etc.
• Decision
• Appropriate therapies
• Drug/Therapy
Detect & Dispatch
• Detection: Early Recognition
• Public education of Stroke sx
• Early access to medical care
• Dispatch: Early EMS and PDI’s
• Caller triage
• EMD recognition of Stroke sx
How to detect?
• Delivery:
• Pre-hospital Transport and
Management
How we scale the pre-
hospital management of the
patient?
CINCINNATI STROKE SCALE
CINCINNATI STROKE SCALE
• Identifies patients with strokes.
• It evaluates three major physical
findings.
•Facial droop
•Motor arm weakness
•Speech abnormalities
CSS - Facial Droop
• Have the patient show their teeth or
smile.
• Normal – both sides of the face move
equally well
• Abnormal – one side of the face does
not move as well as the other side
Arm Drift
• Have the patient close his/her eyes and
hold both arms out.
• Normal – both arms move the same way,
or both arms do not move at all.
• Abnormal – one arm does not move or
one arm drifts down compared to the
other arm.
Other findings such as pronator grip, may
be helpful
Speech
• Have the patient say “You can’t teach
an old dog new tricks.” –
“Perlambagaan Malaysia”
• Normal – patient uses correct words
with no slurring.
• Abnormal – patient slurs words, uses
inappropriate words, or is unable to
speak
“You can’t teach an old
dog new tricks.”
Cincinnati Prehospital
Stroke Scale
• Patients with 1 of these 3 findings -as a
new event - have a 72% probability of an
ischemic stroke.
• If all 3 findings are present the probability
of an acute stroke is more than 85%
• Immediately contact medical control and
the destination ED and provide prearrival
notification.
Stroke – Management
In Review:
Prehospital Critical Actions
• Assess and support cardiorespiratory function
• Assess and support blood glucose
• Assess and support oxygenation and ventilation
• Assess neurologic function
• Determine precise time of symptom onset
• Determine essential medical information
• Provide rapid emergent transport to ED
• Notify ED that a possible stroke patient is en route
Stroke - Management
• Door: ER Triage
• Stroke evaluation targets for stroke patients who are thrombolytic
candidates
Door-to–doctor first sees patient …….………… 10 min
Door-to–CT completed …….………………….. 25 min
Door-to–CT read ...…………..………………… 45 min
Door-to–fibrinolytic therapy starts …………….. 60 min
Neurologic expertise available* …..…………… 15 min
Neurosurgical expertise available* …………… 2 hours
Admitted to monitored bed ..……...…………… 3 hours
*By phone or in person
Stroke - Management
• Data: ER Evaluation and Management
• Assessment Goal: in first 10 minutes
• Assess ABCs, vital signs
• Provide oxygen by nasal cannula
• Obtain IV access; obtain blood samples (CBC, ’lytes, coagulation
studies)
• Obtain 12-lead ECG, check rhythm, place on monitor
• Check blood sugar; treat if indicated
• Alert Stroke Team: neurologist, radiologist, CT technician
• Perform general neurologic screening assessment
Stroke - Management
Assessment Goal: in first 25 minutes
• Review patient history
• Establish symptom onset (<6 hours required for fibrinolytics)
• Perform physical examination
• Perform neurologic exam
• Determine level of consciousness (Glasgow Coma Scale)
• Determine level of stroke severity (NIHSS or Hunt and Hess Scale)
• Order urgent non-contrast CT scan/angiogram if non-hemorrhage
(door-to–CT scan performed: goal <25 min from arrival)
• Read CT scan (door-to–CT read: goal <45 min from arrival)
• Perform lateral cervical spine x-ray (if patient comatose/trauma
history)
Stroke - Management
• ER Diagnostic Studies
• CT scan – done w/in 25 mins, read w/in 45 mins
• r/o hemorrhage
• Often normal early in ischemic stroke
• Lumbar puncture
• EKG
• Changes may be caused by or cause of stroke
• MRA (Magnetic Resonance Angiography)
• Cerebral Angiography
Hypodense area:
• Ischemic area with
edema, swelling
• Indicates >3 hours old
• No fibrinolytics!
(White areas indicate
hyperdensity = blood)
Large left frontal
intracerebral
hemorrhage.
Intraventricular
bleeding
is also present
No fibrinolytics!
Acute
subarachnoid
hemorrhage
Diffuse areas of white
(hyperdense) images
Blood visible in
ventricles
and multiple areas on
surface of brain
Stroke - Management
• Decision: Specific Therapies
• General Care
• ABC’s, O2
• IV w/ BSS
• Treat hypotension
• Avoid over-hydration
• Monitor input/output
• Normalize BGL
• Manage Elevated BP?
Stroke - Management
• Indications for Antihypertensive therapy
In general:
• Consider: absolute level of BP?
• If BP: >185/>110 mm Hg = fibrinolytic therapy contraindicated
• Consider: other than BP, is patient candidate for fibrinolytics?
• If patient is candidate for fibrinolytics: treat initial
BP >185/>110 mm Hg
• Consider: response to initial efforts to lower BP in ED?
• If treatment brings BP down to <185/110 mm Hg: give fibrinolytics
• Consider: ischemic vs hemorrhagic stroke?
• Treat BP in the 180-230/110-140 mm Hg range the same
• The obvious: no fibrinolytics for hemorrhagic stroke
Stroke - Management
• Decision: Specific Therapies (cont.)
• Management of Seizures
• Benzodiazepines
• Long-acting anticonvulsants
• Management of Increased ICP
• Maintain PaCO2 30mm Hg
• Mannitol/Diuretics
• Barbiturates
• Neurosurgical decompression
Stroke - Management
• Drugs: Thrombolytic Therapy
• Fibrinolytic Therapy Checklist Ischemic Stroke
Candidates for Neurointerventional Therapy
 Age 18 years or older
 Acute signs and symptoms of CVA <6 hours onset.
 No contraindications.
Stroke - Management
Contraindications for Interventional Therapy
Absolute
 Evidence of intracranial hemorrhage on non-contrast head CT
 Patient with early infarct signs on CT scan.
Relative
 Recent (w/in 2 mo’s) cranial or spinal surgery, trauma, or injury
 Known bleeding disorder and/or risk of bleeding including:
- Current anticoagulant therapy, prothrombin time >15 sec.
- Heparin within 48 hrs of admission, PTT elevated
- Platelet count <100,000/mm
 Active internal bleeding w/in the previous 10 days
 Known or suspected pregnancy
 History of stroke w/in past 6 weeks
Stroke - Management
Contraindications for Interventional Therapy (cont.)
Relative
 Patient comatose
 >85 years old
 Diabetic hemorrhagic retinopathy or other opthalmic
hemorrhagic disorder
 Advanced liver or kidney disease
 Other pathology with a propensity for bleeding
 Infectiouse endocarditis
 Severe EKG disturbance, uncontrolled angina or acute MI
Stroke - Management
• Thrombolytic Agents
• TPA
• NINDS trial
• Streptokinase
• VEGGIE trial
• Anticoagulant Therapy
• Heparin
• ASA/Warfarin/Ticlodipine
Stroke - Management
• Management of Hemorrhagic Stroke
• Subarachnoid
• Neurosurgical intervention
• Nimodipine
• Intracerebral
• Management of ICP
• Neurosurgical decompression
• Cerebellar
• Surgical evacuation
• Often associated with good outcome
• Lobar
• Surgical evacuation
14
1
THE END
If they come to you
with stroke, what
investigation what
you like to do?
QUIZ ON
STROKE
MOHD HANAFI RAMLEE
ANSWER
GILIR2 TAU
QUESTION 1
On the history of a patient with suspected or known stroke, what
should you ask about?
ANSWER 1
 Ask about the presenting symptoms of stroke
initially, for example
1. Unilateral weakness or clumsiness
2. Difficulty understanding or expressing spoken
language
3. Altered sensation unilaterally
4. Partial or complete loss of vision in one eye
ANSWER 1
 Questions to focus on the causes of stroke
1. Atherosclerosis, e.g. enquire about vascular risk factors (smoking,
DM, hyperlipids, PVDs, etc…)
2. History of heart disease, e.g. recent myocardial infarction, history
of AF requiring anticoagulation, palpitations
3. History of hypertension (lacunar infarcts due to arteriosclerosis of
small penetrating arteries of the brain)
4. Migraine
5. Manipulation of neck (precipitating cause for dissection of carotid
artery or vertebral artery)
6. Any recent cessation of anticoagulation
7. Family history of stroke
8. History of connective tissue disease (e.g. SLE, vasculitis, etc…)
ANSWER 1
1. History of connective tissue disease (e.g. SLE, vasculitis, etc…)
 Medication history, esp. those that increase risk of stroke include
oral contraceptives, some antihypertensives
 Ask about alcohol consumption and recent falls (may have caused
an intracranial haemorrhage)
 Enquire about premorbid as well as the current level of
independence and mobility
 If patient is incapacitated, ask about social support available at
home
 Don’t forget to screen for depression
QUESTION 2
On examination, what features should you be looking for?
ANSWER 2
A complete examination of the neurological and cardiovascular systems is essential
 Check the fundi for evidence of emboli, hypertensive changes, diabetic changes
and ischaemic neuropathy
 Test the visual fields for homonymous hemianopia
 Listen for bruit over the carotids and orbits (commonly heard in the side
opposite the carotid occlusion, due to increased contralateral flow)
 Decide whether patient is in AF
 Assess BP and test for postural drop
 Listen for murmurs (e.g. AS, infective endocarditis, rheumatic heart disease or
prosthetic valve)
 Note presence of electronic pace maker and assess whether it’s working
 Perform peripheral vascular examination
 Look for complications, e.g. pressure sores, limb contractures and disuse
atrophy of the paralysed limbs
 See if patient has a percutaneous gastrostomy (PEG) feeding tube inserted and,
if present, inspect for cellulitis or pus around the insertion site
QUESTION 3
What clinical features are suggestive of a carotid arterial stroke?
ANSWER 3
 Transient aphasia
 Ipsilateral amaurosis fugax
 Contralateral hemiplegia/monoplegia
 Contralateral hemi or monoparesis
 Carotid bruits (a/w >50% stenosis)
QUESTION 4
What clinical features are suggestive of vertebrobasilar insufficiency?
ANSWER 4
All the Ds
 Dizziness (vertigo and ataxia)
 Diplopia +/- blindness
 Dysphagia
 Dysarthria
 ‘Demi-anaesthesia’ – ipsilateral face, contralateral limb
 Quadraparesis – indicates basilar artery involvement
QUESTION 5
What clinical features are suggestive of a middle cerebral artery
stroke?
ANSWER 5
 Contralateral hemiplegia, hemiparesis, homonymous hemianopia
 UMN face, arm > leg, eyes deviated to side of lesion
 Aphasia (if dominant lobe)
1. Expressive dysphasia, arm + leg involvement – anterior MCA
2. Receptive dysphasia, visual fields defects – posterior MCA
 If non-dominant – speech + comprehension intact
QUESTION 6
What clinical features are suggestive of a posterior cerebral artery
stroke?
ANSWER 6
 Homonymous hemianopia (complete)
 Cortical blindness
 Hemisensory loss
 Ipsilateral III nerve palsy
QUESTION 7
What clinical features are suggestive of an anterior cerebral artery
stroke?
ANSWER 7
 Contralateral monoplegia (UMN)
 Confusion, behaviour disturbance
 Grasp reflex
 Urinary incontinence
QUESTION 8
What clinical features are suggestive of a lacunar syndrome?
ANSWER 8
 Clumsy hand/dysarthria syndrome – lesion in mid pons
 Leg paresis + ataxia – pons or internal capsule
 Pure sensory stroke – usually thalamic
 Pure motor stroke (arm>leg) - pons or internal capsule
QUESTION 9
What investigations would you perform in a stroke patient? – MAINLY
TO DIAGNOSE ANY STROKE….
ANSWER 9
CT or MRI of the head, looking for ischaemic infarcts, haemorrhage, or
mass lesions
QUESTION 10
What investigations would you perform in a stroke patient? MAINLY
TO FIND THE CAUSE OF STROKE
ANSWER 10
 Doppler scan of the carotid arteries – if duplex scan suggest
significant carotid stenosis, esp. in patients <75, ask for results of
carotid angiography, carotid digital substraction angiography or MR
angiography
 If patient is in AF, ask for results of transoesophageal echo, looking
for thrombus or spontaneous atheromatous plaques in ascending
aorta and arch of aorta that may have contributed to stroke
 ECG for AMI and AF
 FBE, looking at Hb (to exclude polycythemia) and platelet count
(rarely, essential thrombocytopenia can contribute to stroke)
 ESR (to exclude an inflammatory arteritic/vasculitic process
 CXR (for cardiomegaly/neoplasm)
 Urea, creatinine and electrolytes
QUESTION 11
What investigations would you perform in a stroke patient?
MAINLY IN YOUNGER PATIENT?
QUESTION 11
 Drug screen, looking for narcotic agents
 Vasculitic screen (if there are features of vasculitis)
 Blood cultures and cardiac imaging if endocarditis is suspected
 Cardiac event monitor looking for paroxysmal AF and
 Thrombophilic screen
WHAT 3 SPECIFIC STRATEGIES IN THE
MANAGEMENT OF ACUTE STROKE THAT HAVE
BEEN PROVEN TO IMPROVE OUTCOMES?
The 3 main strategies are
 Administration of iv tissue plasminogen activator (tPA) within 3hrs
of stroke onset
 Giving aspirin (100mg) within first 48hrs of ischaemic stroke
 Managing patient in a stroke unit
Note that tPA and aspirin are never given before brain imaging to
exclude intracranial haemorrhage
WHAT ARE THE STRATEGIES FOR
SECONDARY PREVENTION OF
STROKE?
There are now at least 4 early strategies for secondary prevention in
improving long term outcomes after TIAs or stoke
 Antiplatelet therapy, e.g. aspirin, clopidogrel, aspirin with
dipyridamole
 Blood pressure lowering
 Warfarin (indicated for patients with AF)
 Carotid endarterectomy (indicated when stenosis >70%)
 Lipids lowering is the fifth strategy to consider
A PATIENT JUST HAD A STROKE, WHAT
CRITERIA DO YOU LOOK FOR TO SELECT THIS
PATIENT FOR REHAB.
1. Patients must be medically stable (i.e. no aspiration pneumonia,
AMI, DVT, etc…)
2. They have a functional disability
3. They have the intellectual capability to learn
4. There are defined goals to be achieved
5. There are non or minimal co-morbidities, e.g. recurrent stroke,
AMI, COAD, etc…
6. They are not clinically depressed (this can affect motivation to
rehabilitate)
7. They didn’t suffer from a dense stroke or ones that causes hemi-
neglect (e.g. a right hemisphere stroke)
WHAT ARE SOME OF THE COMMON
PREDICTORS OF POOR FUNCTIONAL
OUTCOME AFTER A STROKE?
 A dense stroke, a recurrent stroke, or a stroke resulting in hemi-
neglect
 Impairment of bladder and bowel function
 Depression or cognitive deficits
 Delayed acute medical care or delayed rehabilitation
 Co-morbidities and poor social support

Más contenido relacionado

La actualidad más candente

NEUROCARDIOGENIC SYNCOPE ppt
NEUROCARDIOGENIC SYNCOPE ppt NEUROCARDIOGENIC SYNCOPE ppt
NEUROCARDIOGENIC SYNCOPE ppt Iqbal Dar
 
Traumatic brain injury and Spinal cord injury
Traumatic brain injury and Spinal cord injuryTraumatic brain injury and Spinal cord injury
Traumatic brain injury and Spinal cord injuryJack Frost
 
Approach to a_case_of_headache
Approach to a_case_of_headacheApproach to a_case_of_headache
Approach to a_case_of_headacheMohit Aggarwal
 
Intracranial space occupying lesions
Intracranial space occupying lesionsIntracranial space occupying lesions
Intracranial space occupying lesionsnoorulain89
 
Stroke (cva) , CVA, Cerebrovascular Accident, Transient Ischemic Attack
Stroke (cva) , CVA, Cerebrovascular Accident, Transient Ischemic AttackStroke (cva) , CVA, Cerebrovascular Accident, Transient Ischemic Attack
Stroke (cva) , CVA, Cerebrovascular Accident, Transient Ischemic AttackDr. Shahbaz Ahmad PT Chaudhary
 
Physiology and mli of shock
Physiology and mli of shockPhysiology and mli of shock
Physiology and mli of shocktsokos
 
Systemic Lupus Erythematosis
Systemic Lupus ErythematosisSystemic Lupus Erythematosis
Systemic Lupus ErythematosisJayakrishnan MP
 
Altered sensorium
Altered sensorium Altered sensorium
Altered sensorium Sudhir Dev
 
Brainsteam stroke by Sunil Kumar Daha
Brainsteam stroke by Sunil Kumar DahaBrainsteam stroke by Sunil Kumar Daha
Brainsteam stroke by Sunil Kumar Dahasunil kumar daha
 
HEADACHE - CLASSIFICATION
HEADACHE - CLASSIFICATIONHEADACHE - CLASSIFICATION
HEADACHE - CLASSIFICATIONRajesh Kabilan
 
Cerebrovascular disease pathology stroke
Cerebrovascular disease pathology strokeCerebrovascular disease pathology stroke
Cerebrovascular disease pathology strokeAppy Akshay Agarwal
 
Cerebrovascular disease (CVA / Stroke)
Cerebrovascular disease (CVA / Stroke)Cerebrovascular disease (CVA / Stroke)
Cerebrovascular disease (CVA / Stroke)Richard Brown
 
Cerebrovascular disease2
Cerebrovascular disease2Cerebrovascular disease2
Cerebrovascular disease2Pratap Tiwari
 
Ischemic Stroke Subclassification, An Asian Viewpoint
Ischemic Stroke Subclassification, An Asian ViewpointIschemic Stroke Subclassification, An Asian Viewpoint
Ischemic Stroke Subclassification, An Asian ViewpointErsifa Fatimah
 

La actualidad más candente (20)

Headache ppt
Headache pptHeadache ppt
Headache ppt
 
NEUROCARDIOGENIC SYNCOPE ppt
NEUROCARDIOGENIC SYNCOPE ppt NEUROCARDIOGENIC SYNCOPE ppt
NEUROCARDIOGENIC SYNCOPE ppt
 
Traumatic brain injury and Spinal cord injury
Traumatic brain injury and Spinal cord injuryTraumatic brain injury and Spinal cord injury
Traumatic brain injury and Spinal cord injury
 
Approach to a_case_of_headache
Approach to a_case_of_headacheApproach to a_case_of_headache
Approach to a_case_of_headache
 
Intracranial space occupying lesions
Intracranial space occupying lesionsIntracranial space occupying lesions
Intracranial space occupying lesions
 
Stroke (cva) , CVA, Cerebrovascular Accident, Transient Ischemic Attack
Stroke (cva) , CVA, Cerebrovascular Accident, Transient Ischemic AttackStroke (cva) , CVA, Cerebrovascular Accident, Transient Ischemic Attack
Stroke (cva) , CVA, Cerebrovascular Accident, Transient Ischemic Attack
 
Physiology and mli of shock
Physiology and mli of shockPhysiology and mli of shock
Physiology and mli of shock
 
Neuropathies & myopathies - an overview
Neuropathies &  myopathies - an overviewNeuropathies &  myopathies - an overview
Neuropathies & myopathies - an overview
 
Systemic Lupus Erythematosis
Systemic Lupus ErythematosisSystemic Lupus Erythematosis
Systemic Lupus Erythematosis
 
Altered sensorium
Altered sensorium Altered sensorium
Altered sensorium
 
Brainsteam stroke by Sunil Kumar Daha
Brainsteam stroke by Sunil Kumar DahaBrainsteam stroke by Sunil Kumar Daha
Brainsteam stroke by Sunil Kumar Daha
 
HEADACHE - CLASSIFICATION
HEADACHE - CLASSIFICATIONHEADACHE - CLASSIFICATION
HEADACHE - CLASSIFICATION
 
Cerebrovascular disease pathology stroke
Cerebrovascular disease pathology strokeCerebrovascular disease pathology stroke
Cerebrovascular disease pathology stroke
 
Compressive Myelopathy
Compressive MyelopathyCompressive Myelopathy
Compressive Myelopathy
 
Cerebrovascular disease (CVA / Stroke)
Cerebrovascular disease (CVA / Stroke)Cerebrovascular disease (CVA / Stroke)
Cerebrovascular disease (CVA / Stroke)
 
Cerebrovascular disease2
Cerebrovascular disease2Cerebrovascular disease2
Cerebrovascular disease2
 
Ischemic Stroke Subclassification, An Asian Viewpoint
Ischemic Stroke Subclassification, An Asian ViewpointIschemic Stroke Subclassification, An Asian Viewpoint
Ischemic Stroke Subclassification, An Asian Viewpoint
 
Headache
HeadacheHeadache
Headache
 
Systemic lupus
Systemic lupusSystemic lupus
Systemic lupus
 
Approach to patient with headache
Approach to patient with headacheApproach to patient with headache
Approach to patient with headache
 

Destacado

Transient ischemic attacks
Transient ischemic attacksTransient ischemic attacks
Transient ischemic attacksNeurologyKota
 
Care of an unconcious pt
Care of an unconcious ptCare of an unconcious pt
Care of an unconcious ptMEEQAT HOSPITAL
 
cerebrovascular accident
cerebrovascular accidentcerebrovascular accident
cerebrovascular accidentgeeta joshi
 
nursing care on unconcious patient
nursing care on unconcious patientnursing care on unconcious patient
nursing care on unconcious patientAdhikari Kirti
 
Parkinson’S Disease
Parkinson’S DiseaseParkinson’S Disease
Parkinson’S Diseaseguest27ee33
 
Acute management of Stroke By Dr Sanjay jaiswal Neurologist sept2012
 Acute  management of Stroke By Dr Sanjay  jaiswal  Neurologist sept2012 Acute  management of Stroke By Dr Sanjay  jaiswal  Neurologist sept2012
Acute management of Stroke By Dr Sanjay jaiswal Neurologist sept2012Sanjay Jaiswal
 
Stroke (cerebrovascular accident)
Stroke (cerebrovascular accident)Stroke (cerebrovascular accident)
Stroke (cerebrovascular accident)Monster Gaga
 
STROKE LECTURE By Arlyn M. Valencia, M.D. Associate Professo University Of Ne...
STROKE LECTURE By Arlyn M. Valencia, M.D. Associate Professo University Of Ne...STROKE LECTURE By Arlyn M. Valencia, M.D. Associate Professo University Of Ne...
STROKE LECTURE By Arlyn M. Valencia, M.D. Associate Professo University Of Ne...Arlyn Valencia, M.D.
 
The Top Skills That Can Get You Hired in 2017
The Top Skills That Can Get You Hired in 2017The Top Skills That Can Get You Hired in 2017
The Top Skills That Can Get You Hired in 2017LinkedIn
 

Destacado (13)

Transient ischemic attacks
Transient ischemic attacksTransient ischemic attacks
Transient ischemic attacks
 
Care of an unconcious pt
Care of an unconcious ptCare of an unconcious pt
Care of an unconcious pt
 
cerebrovascular accident
cerebrovascular accidentcerebrovascular accident
cerebrovascular accident
 
Six stroke engine ppt
Six stroke engine pptSix stroke engine ppt
Six stroke engine ppt
 
nursing care on unconcious patient
nursing care on unconcious patientnursing care on unconcious patient
nursing care on unconcious patient
 
Parkinson s disease
Parkinson s diseaseParkinson s disease
Parkinson s disease
 
Parkinson’S Disease
Parkinson’S DiseaseParkinson’S Disease
Parkinson’S Disease
 
Acute management of Stroke By Dr Sanjay jaiswal Neurologist sept2012
 Acute  management of Stroke By Dr Sanjay  jaiswal  Neurologist sept2012 Acute  management of Stroke By Dr Sanjay  jaiswal  Neurologist sept2012
Acute management of Stroke By Dr Sanjay jaiswal Neurologist sept2012
 
Stroke (cerebrovascular accident)
Stroke (cerebrovascular accident)Stroke (cerebrovascular accident)
Stroke (cerebrovascular accident)
 
Myasthenia gravis
Myasthenia gravisMyasthenia gravis
Myasthenia gravis
 
STROKE LECTURE By Arlyn M. Valencia, M.D. Associate Professo University Of Ne...
STROKE LECTURE By Arlyn M. Valencia, M.D. Associate Professo University Of Ne...STROKE LECTURE By Arlyn M. Valencia, M.D. Associate Professo University Of Ne...
STROKE LECTURE By Arlyn M. Valencia, M.D. Associate Professo University Of Ne...
 
Stroke
StrokeStroke
Stroke
 
The Top Skills That Can Get You Hired in 2017
The Top Skills That Can Get You Hired in 2017The Top Skills That Can Get You Hired in 2017
The Top Skills That Can Get You Hired in 2017
 

Similar a Stroke [uncensored] - by MHR Corporation

Clinical aspects of the nervous system
Clinical aspects of the nervous systemClinical aspects of the nervous system
Clinical aspects of the nervous systemvajira54
 
Ischemic stroke -Basics
Ischemic stroke -Basics Ischemic stroke -Basics
Ischemic stroke -Basics Mohamad Yaakub
 
stroke syndrome 2.pdf
stroke syndrome 2.pdfstroke syndrome 2.pdf
stroke syndrome 2.pdfssuser5d654b
 
Approach to TIA/ CVA
Approach to TIA/ CVAApproach to TIA/ CVA
Approach to TIA/ CVAAhmad Shahir
 
Final [CH13] NOTES ppt, Neurological Problems.ppt
Final [CH13] NOTES ppt, Neurological Problems.pptFinal [CH13] NOTES ppt, Neurological Problems.ppt
Final [CH13] NOTES ppt, Neurological Problems.pptTristanBabaylan1
 
CLINICAL FEATURES, INVESTIGATIONS AND PROGNOSIS OF BRAIN SEMINAR 2.pptx
CLINICAL FEATURES, INVESTIGATIONS AND PROGNOSIS OF BRAIN SEMINAR 2.pptxCLINICAL FEATURES, INVESTIGATIONS AND PROGNOSIS OF BRAIN SEMINAR 2.pptx
CLINICAL FEATURES, INVESTIGATIONS AND PROGNOSIS OF BRAIN SEMINAR 2.pptxcheshtasharma22
 
Stroke and cerebrovascular accident
Stroke and cerebrovascular accidentStroke and cerebrovascular accident
Stroke and cerebrovascular accidentDr. Rubz
 
Stroke and its management
Stroke and its managementStroke and its management
Stroke and its managementDr. Ankit Gaur
 
Ataxia 130514030409-phpapp01
Ataxia 130514030409-phpapp01Ataxia 130514030409-phpapp01
Ataxia 130514030409-phpapp01Indhu Reddy
 
Brief review in cerebellar stroke -diagnosis
Brief review in cerebellar stroke -diagnosisBrief review in cerebellar stroke -diagnosis
Brief review in cerebellar stroke -diagnosisKaminiVinathan1
 
Types of cva and stroke syndromes
Types of cva and stroke syndromesTypes of cva and stroke syndromes
Types of cva and stroke syndromesDarendrajit Longjam
 
Approach to a_patient_presenting_with_hemiplegia
Approach to a_patient_presenting_with_hemiplegiaApproach to a_patient_presenting_with_hemiplegia
Approach to a_patient_presenting_with_hemiplegiaalyaqdhan
 

Similar a Stroke [uncensored] - by MHR Corporation (20)

Stroke localization
Stroke localizationStroke localization
Stroke localization
 
Clinical aspects of the nervous system
Clinical aspects of the nervous systemClinical aspects of the nervous system
Clinical aspects of the nervous system
 
Stroke
Stroke Stroke
Stroke
 
Ischemic stroke -Basics
Ischemic stroke -Basics Ischemic stroke -Basics
Ischemic stroke -Basics
 
stroke syndrome 2.pdf
stroke syndrome 2.pdfstroke syndrome 2.pdf
stroke syndrome 2.pdf
 
Approach to TIA/ CVA
Approach to TIA/ CVAApproach to TIA/ CVA
Approach to TIA/ CVA
 
Final [CH13] NOTES ppt, Neurological Problems.ppt
Final [CH13] NOTES ppt, Neurological Problems.pptFinal [CH13] NOTES ppt, Neurological Problems.ppt
Final [CH13] NOTES ppt, Neurological Problems.ppt
 
CLINICAL FEATURES, INVESTIGATIONS AND PROGNOSIS OF BRAIN SEMINAR 2.pptx
CLINICAL FEATURES, INVESTIGATIONS AND PROGNOSIS OF BRAIN SEMINAR 2.pptxCLINICAL FEATURES, INVESTIGATIONS AND PROGNOSIS OF BRAIN SEMINAR 2.pptx
CLINICAL FEATURES, INVESTIGATIONS AND PROGNOSIS OF BRAIN SEMINAR 2.pptx
 
Stroke and cerebrovascular accident
Stroke and cerebrovascular accidentStroke and cerebrovascular accident
Stroke and cerebrovascular accident
 
Stroke and its management
Stroke and its managementStroke and its management
Stroke and its management
 
Ataxia 130514030409-phpapp01
Ataxia 130514030409-phpapp01Ataxia 130514030409-phpapp01
Ataxia 130514030409-phpapp01
 
Ataxia seminar
Ataxia seminarAtaxia seminar
Ataxia seminar
 
Ataxia shyam
Ataxia shyamAtaxia shyam
Ataxia shyam
 
Central vertigo
Central vertigoCentral vertigo
Central vertigo
 
Brief review in cerebellar stroke -diagnosis
Brief review in cerebellar stroke -diagnosisBrief review in cerebellar stroke -diagnosis
Brief review in cerebellar stroke -diagnosis
 
Ataxia
AtaxiaAtaxia
Ataxia
 
OT for cva
OT for cvaOT for cva
OT for cva
 
Types of cva and stroke syndromes
Types of cva and stroke syndromesTypes of cva and stroke syndromes
Types of cva and stroke syndromes
 
Approach to a_patient_presenting_with_hemiplegia
Approach to a_patient_presenting_with_hemiplegiaApproach to a_patient_presenting_with_hemiplegia
Approach to a_patient_presenting_with_hemiplegia
 
Gait disorders
Gait disordersGait disorders
Gait disorders
 

Más de Mohd Hanafi

Oncologic emergencies
Oncologic emergenciesOncologic emergencies
Oncologic emergenciesMohd Hanafi
 
Practice makes perfect
Practice makes perfectPractice makes perfect
Practice makes perfectMohd Hanafi
 
Osce ear nose n telinga
Osce ear nose n telingaOsce ear nose n telinga
Osce ear nose n telingaMohd Hanafi
 
Pathological findings
Pathological findingsPathological findings
Pathological findingsMohd Hanafi
 
Dengue algorithm
Dengue algorithmDengue algorithm
Dengue algorithmMohd Hanafi
 
Clinical approach to the floppy child
Clinical approach to the floppy childClinical approach to the floppy child
Clinical approach to the floppy childMohd Hanafi
 
approach to the unknown rash
approach to the unknown rashapproach to the unknown rash
approach to the unknown rashMohd Hanafi
 
Mr. Kerengga [CPC]
Mr. Kerengga [CPC]Mr. Kerengga [CPC]
Mr. Kerengga [CPC]Mohd Hanafi
 
Antibiotics by class
Antibiotics by classAntibiotics by class
Antibiotics by classMohd Hanafi
 
Gestational Diabetes Mellitus - DevanRaj
Gestational Diabetes Mellitus - DevanRajGestational Diabetes Mellitus - DevanRaj
Gestational Diabetes Mellitus - DevanRajMohd Hanafi
 
Pyrexia of unknown origin (puo)
Pyrexia of unknown origin (puo)Pyrexia of unknown origin (puo)
Pyrexia of unknown origin (puo)Mohd Hanafi
 
Malpresentation illi(2)
Malpresentation illi(2)Malpresentation illi(2)
Malpresentation illi(2)Mohd Hanafi
 
4 normal labour and delivery
4 normal labour and delivery4 normal labour and delivery
4 normal labour and deliveryMohd Hanafi
 
Postpartum haemorrhage
Postpartum haemorrhagePostpartum haemorrhage
Postpartum haemorrhageMohd Hanafi
 

Más de Mohd Hanafi (20)

Oncologic emergencies
Oncologic emergenciesOncologic emergencies
Oncologic emergencies
 
Practice makes perfect
Practice makes perfectPractice makes perfect
Practice makes perfect
 
Osce cuckoos
Osce cuckoosOsce cuckoos
Osce cuckoos
 
houskee
houskeehouskee
houskee
 
Osce ear nose n telinga
Osce ear nose n telingaOsce ear nose n telinga
Osce ear nose n telinga
 
Funduscopy
FunduscopyFunduscopy
Funduscopy
 
Eye osce
Eye osceEye osce
Eye osce
 
Pathological findings
Pathological findingsPathological findings
Pathological findings
 
Dengue algorithm
Dengue algorithmDengue algorithm
Dengue algorithm
 
Amenorrhea
AmenorrheaAmenorrhea
Amenorrhea
 
Clinical approach to the floppy child
Clinical approach to the floppy childClinical approach to the floppy child
Clinical approach to the floppy child
 
approach to the unknown rash
approach to the unknown rashapproach to the unknown rash
approach to the unknown rash
 
Mr. Kerengga [CPC]
Mr. Kerengga [CPC]Mr. Kerengga [CPC]
Mr. Kerengga [CPC]
 
Antibiotics by class
Antibiotics by classAntibiotics by class
Antibiotics by class
 
Gestational Diabetes Mellitus - DevanRaj
Gestational Diabetes Mellitus - DevanRajGestational Diabetes Mellitus - DevanRaj
Gestational Diabetes Mellitus - DevanRaj
 
Pyrexia of unknown origin (puo)
Pyrexia of unknown origin (puo)Pyrexia of unknown origin (puo)
Pyrexia of unknown origin (puo)
 
Malpresentation illi(2)
Malpresentation illi(2)Malpresentation illi(2)
Malpresentation illi(2)
 
13 partogram
13 partogram13 partogram
13 partogram
 
4 normal labour and delivery
4 normal labour and delivery4 normal labour and delivery
4 normal labour and delivery
 
Postpartum haemorrhage
Postpartum haemorrhagePostpartum haemorrhage
Postpartum haemorrhage
 

Último

Call Girls Kanakapura Road Just Call 7001305949 Top Class Call Girl Service A...
Call Girls Kanakapura Road Just Call 7001305949 Top Class Call Girl Service A...Call Girls Kanakapura Road Just Call 7001305949 Top Class Call Girl Service A...
Call Girls Kanakapura Road Just Call 7001305949 Top Class Call Girl Service A...narwatsonia7
 
Hemostasis Physiology and Clinical correlations by Dr Faiza.pdf
Hemostasis Physiology and Clinical correlations by Dr Faiza.pdfHemostasis Physiology and Clinical correlations by Dr Faiza.pdf
Hemostasis Physiology and Clinical correlations by Dr Faiza.pdfMedicoseAcademics
 
Call Girls Service in Bommanahalli - 7001305949 with real photos and phone nu...
Call Girls Service in Bommanahalli - 7001305949 with real photos and phone nu...Call Girls Service in Bommanahalli - 7001305949 with real photos and phone nu...
Call Girls Service in Bommanahalli - 7001305949 with real photos and phone nu...narwatsonia7
 
Low Rate Call Girls Mumbai Suman 9910780858 Independent Escort Service Mumbai
Low Rate Call Girls Mumbai Suman 9910780858 Independent Escort Service MumbaiLow Rate Call Girls Mumbai Suman 9910780858 Independent Escort Service Mumbai
Low Rate Call Girls Mumbai Suman 9910780858 Independent Escort Service Mumbaisonalikaur4
 
Case Report Peripartum Cardiomyopathy.pptx
Case Report Peripartum Cardiomyopathy.pptxCase Report Peripartum Cardiomyopathy.pptx
Case Report Peripartum Cardiomyopathy.pptxNiranjan Chavan
 
Statistical modeling in pharmaceutical research and development.
Statistical modeling in pharmaceutical research and development.Statistical modeling in pharmaceutical research and development.
Statistical modeling in pharmaceutical research and development.ANJALI
 
Call Girls Hsr Layout Just Call 7001305949 Top Class Call Girl Service Available
Call Girls Hsr Layout Just Call 7001305949 Top Class Call Girl Service AvailableCall Girls Hsr Layout Just Call 7001305949 Top Class Call Girl Service Available
Call Girls Hsr Layout Just Call 7001305949 Top Class Call Girl Service Availablenarwatsonia7
 
VIP Call Girls Lucknow Nandini 7001305949 Independent Escort Service Lucknow
VIP Call Girls Lucknow Nandini 7001305949 Independent Escort Service LucknowVIP Call Girls Lucknow Nandini 7001305949 Independent Escort Service Lucknow
VIP Call Girls Lucknow Nandini 7001305949 Independent Escort Service Lucknownarwatsonia7
 
See the 2,456 pharmacies on the National E-Pharmacy Platform
See the 2,456 pharmacies on the National E-Pharmacy PlatformSee the 2,456 pharmacies on the National E-Pharmacy Platform
See the 2,456 pharmacies on the National E-Pharmacy PlatformKweku Zurek
 
Call Girls Viman Nagar 7001305949 All Area Service COD available Any Time
Call Girls Viman Nagar 7001305949 All Area Service COD available Any TimeCall Girls Viman Nagar 7001305949 All Area Service COD available Any Time
Call Girls Viman Nagar 7001305949 All Area Service COD available Any Timevijaych2041
 
Hematology and Immunology - Leukocytes Functions
Hematology and Immunology - Leukocytes FunctionsHematology and Immunology - Leukocytes Functions
Hematology and Immunology - Leukocytes FunctionsMedicoseAcademics
 
Kolkata Call Girls Services 9907093804 @24x7 High Class Babes Here Call Now
Kolkata Call Girls Services 9907093804 @24x7 High Class Babes Here Call NowKolkata Call Girls Services 9907093804 @24x7 High Class Babes Here Call Now
Kolkata Call Girls Services 9907093804 @24x7 High Class Babes Here Call NowNehru place Escorts
 
Call Girls Jp Nagar Just Call 7001305949 Top Class Call Girl Service Available
Call Girls Jp Nagar Just Call 7001305949 Top Class Call Girl Service AvailableCall Girls Jp Nagar Just Call 7001305949 Top Class Call Girl Service Available
Call Girls Jp Nagar Just Call 7001305949 Top Class Call Girl Service Availablenarwatsonia7
 
Housewife Call Girls Bangalore - Call 7001305949 Rs-3500 with A/C Room Cash o...
Housewife Call Girls Bangalore - Call 7001305949 Rs-3500 with A/C Room Cash o...Housewife Call Girls Bangalore - Call 7001305949 Rs-3500 with A/C Room Cash o...
Housewife Call Girls Bangalore - Call 7001305949 Rs-3500 with A/C Room Cash o...narwatsonia7
 
Call Girls Frazer Town Just Call 7001305949 Top Class Call Girl Service Avail...
Call Girls Frazer Town Just Call 7001305949 Top Class Call Girl Service Avail...Call Girls Frazer Town Just Call 7001305949 Top Class Call Girl Service Avail...
Call Girls Frazer Town Just Call 7001305949 Top Class Call Girl Service Avail...narwatsonia7
 
Call Girls Thane Just Call 9910780858 Get High Class Call Girls Service
Call Girls Thane Just Call 9910780858 Get High Class Call Girls ServiceCall Girls Thane Just Call 9910780858 Get High Class Call Girls Service
Call Girls Thane Just Call 9910780858 Get High Class Call Girls Servicesonalikaur4
 
Call Girls Hebbal Just Call 7001305949 Top Class Call Girl Service Available
Call Girls Hebbal Just Call 7001305949 Top Class Call Girl Service AvailableCall Girls Hebbal Just Call 7001305949 Top Class Call Girl Service Available
Call Girls Hebbal Just Call 7001305949 Top Class Call Girl Service Availablenarwatsonia7
 
Book Call Girls in Kasavanahalli - 7001305949 with real photos and phone numbers
Book Call Girls in Kasavanahalli - 7001305949 with real photos and phone numbersBook Call Girls in Kasavanahalli - 7001305949 with real photos and phone numbers
Book Call Girls in Kasavanahalli - 7001305949 with real photos and phone numbersnarwatsonia7
 
Noida Sector 135 Call Girls ( 9873940964 ) Book Hot And Sexy Girls In A Few C...
Noida Sector 135 Call Girls ( 9873940964 ) Book Hot And Sexy Girls In A Few C...Noida Sector 135 Call Girls ( 9873940964 ) Book Hot And Sexy Girls In A Few C...
Noida Sector 135 Call Girls ( 9873940964 ) Book Hot And Sexy Girls In A Few C...rajnisinghkjn
 
Air-Hostess Call Girls Madambakkam - Phone No 7001305949 For Ultimate Sexual ...
Air-Hostess Call Girls Madambakkam - Phone No 7001305949 For Ultimate Sexual ...Air-Hostess Call Girls Madambakkam - Phone No 7001305949 For Ultimate Sexual ...
Air-Hostess Call Girls Madambakkam - Phone No 7001305949 For Ultimate Sexual ...Ahmedabad Escorts
 

Último (20)

Call Girls Kanakapura Road Just Call 7001305949 Top Class Call Girl Service A...
Call Girls Kanakapura Road Just Call 7001305949 Top Class Call Girl Service A...Call Girls Kanakapura Road Just Call 7001305949 Top Class Call Girl Service A...
Call Girls Kanakapura Road Just Call 7001305949 Top Class Call Girl Service A...
 
Hemostasis Physiology and Clinical correlations by Dr Faiza.pdf
Hemostasis Physiology and Clinical correlations by Dr Faiza.pdfHemostasis Physiology and Clinical correlations by Dr Faiza.pdf
Hemostasis Physiology and Clinical correlations by Dr Faiza.pdf
 
Call Girls Service in Bommanahalli - 7001305949 with real photos and phone nu...
Call Girls Service in Bommanahalli - 7001305949 with real photos and phone nu...Call Girls Service in Bommanahalli - 7001305949 with real photos and phone nu...
Call Girls Service in Bommanahalli - 7001305949 with real photos and phone nu...
 
Low Rate Call Girls Mumbai Suman 9910780858 Independent Escort Service Mumbai
Low Rate Call Girls Mumbai Suman 9910780858 Independent Escort Service MumbaiLow Rate Call Girls Mumbai Suman 9910780858 Independent Escort Service Mumbai
Low Rate Call Girls Mumbai Suman 9910780858 Independent Escort Service Mumbai
 
Case Report Peripartum Cardiomyopathy.pptx
Case Report Peripartum Cardiomyopathy.pptxCase Report Peripartum Cardiomyopathy.pptx
Case Report Peripartum Cardiomyopathy.pptx
 
Statistical modeling in pharmaceutical research and development.
Statistical modeling in pharmaceutical research and development.Statistical modeling in pharmaceutical research and development.
Statistical modeling in pharmaceutical research and development.
 
Call Girls Hsr Layout Just Call 7001305949 Top Class Call Girl Service Available
Call Girls Hsr Layout Just Call 7001305949 Top Class Call Girl Service AvailableCall Girls Hsr Layout Just Call 7001305949 Top Class Call Girl Service Available
Call Girls Hsr Layout Just Call 7001305949 Top Class Call Girl Service Available
 
VIP Call Girls Lucknow Nandini 7001305949 Independent Escort Service Lucknow
VIP Call Girls Lucknow Nandini 7001305949 Independent Escort Service LucknowVIP Call Girls Lucknow Nandini 7001305949 Independent Escort Service Lucknow
VIP Call Girls Lucknow Nandini 7001305949 Independent Escort Service Lucknow
 
See the 2,456 pharmacies on the National E-Pharmacy Platform
See the 2,456 pharmacies on the National E-Pharmacy PlatformSee the 2,456 pharmacies on the National E-Pharmacy Platform
See the 2,456 pharmacies on the National E-Pharmacy Platform
 
Call Girls Viman Nagar 7001305949 All Area Service COD available Any Time
Call Girls Viman Nagar 7001305949 All Area Service COD available Any TimeCall Girls Viman Nagar 7001305949 All Area Service COD available Any Time
Call Girls Viman Nagar 7001305949 All Area Service COD available Any Time
 
Hematology and Immunology - Leukocytes Functions
Hematology and Immunology - Leukocytes FunctionsHematology and Immunology - Leukocytes Functions
Hematology and Immunology - Leukocytes Functions
 
Kolkata Call Girls Services 9907093804 @24x7 High Class Babes Here Call Now
Kolkata Call Girls Services 9907093804 @24x7 High Class Babes Here Call NowKolkata Call Girls Services 9907093804 @24x7 High Class Babes Here Call Now
Kolkata Call Girls Services 9907093804 @24x7 High Class Babes Here Call Now
 
Call Girls Jp Nagar Just Call 7001305949 Top Class Call Girl Service Available
Call Girls Jp Nagar Just Call 7001305949 Top Class Call Girl Service AvailableCall Girls Jp Nagar Just Call 7001305949 Top Class Call Girl Service Available
Call Girls Jp Nagar Just Call 7001305949 Top Class Call Girl Service Available
 
Housewife Call Girls Bangalore - Call 7001305949 Rs-3500 with A/C Room Cash o...
Housewife Call Girls Bangalore - Call 7001305949 Rs-3500 with A/C Room Cash o...Housewife Call Girls Bangalore - Call 7001305949 Rs-3500 with A/C Room Cash o...
Housewife Call Girls Bangalore - Call 7001305949 Rs-3500 with A/C Room Cash o...
 
Call Girls Frazer Town Just Call 7001305949 Top Class Call Girl Service Avail...
Call Girls Frazer Town Just Call 7001305949 Top Class Call Girl Service Avail...Call Girls Frazer Town Just Call 7001305949 Top Class Call Girl Service Avail...
Call Girls Frazer Town Just Call 7001305949 Top Class Call Girl Service Avail...
 
Call Girls Thane Just Call 9910780858 Get High Class Call Girls Service
Call Girls Thane Just Call 9910780858 Get High Class Call Girls ServiceCall Girls Thane Just Call 9910780858 Get High Class Call Girls Service
Call Girls Thane Just Call 9910780858 Get High Class Call Girls Service
 
Call Girls Hebbal Just Call 7001305949 Top Class Call Girl Service Available
Call Girls Hebbal Just Call 7001305949 Top Class Call Girl Service AvailableCall Girls Hebbal Just Call 7001305949 Top Class Call Girl Service Available
Call Girls Hebbal Just Call 7001305949 Top Class Call Girl Service Available
 
Book Call Girls in Kasavanahalli - 7001305949 with real photos and phone numbers
Book Call Girls in Kasavanahalli - 7001305949 with real photos and phone numbersBook Call Girls in Kasavanahalli - 7001305949 with real photos and phone numbers
Book Call Girls in Kasavanahalli - 7001305949 with real photos and phone numbers
 
Noida Sector 135 Call Girls ( 9873940964 ) Book Hot And Sexy Girls In A Few C...
Noida Sector 135 Call Girls ( 9873940964 ) Book Hot And Sexy Girls In A Few C...Noida Sector 135 Call Girls ( 9873940964 ) Book Hot And Sexy Girls In A Few C...
Noida Sector 135 Call Girls ( 9873940964 ) Book Hot And Sexy Girls In A Few C...
 
Air-Hostess Call Girls Madambakkam - Phone No 7001305949 For Ultimate Sexual ...
Air-Hostess Call Girls Madambakkam - Phone No 7001305949 For Ultimate Sexual ...Air-Hostess Call Girls Madambakkam - Phone No 7001305949 For Ultimate Sexual ...
Air-Hostess Call Girls Madambakkam - Phone No 7001305949 For Ultimate Sexual ...
 

Stroke [uncensored] - by MHR Corporation

  • 2. Definition • Transient Ischaemic Attack (TIA) – an acute focal neurological deficit resulting from cerebrovascular disease with resolution of signs and symptoms within 24 hours. • Reversible Ischaemic Neurological Deficit (RIND) – attack lasting longer than 24 hours but with complete clearance of signs and symptoms within 7 days. • Completed Stroke – neurological deficit lasts longer than 7 days.
  • 3. HOW YOU GIVE THE DIAGNOSIS? • The diagnosis should provide answers to the following questions: • 1. What is the neurological deficit? • 2. Where is the lesion? • 3. What is the lesion? • 4. Why has the lesion occurred? • 5. What are the potential complications and prognosis?
  • 4. What Artery Involved? • Aphasia (dominant hemisphere) • Hemiparesis / plegia • Hemisensory loss/disturbance • Homonymous hemianopia • Parietal lobe dysfunction, e.g. astereognosis, agraphaesthesia, impaired two-point discrimination, sensory and visual inattention, left-right dissociation and acalculia
  • 5. What Artery Involved? • Weakness of lower limb more than upper limb
  • 6. What Artery Involved? • Homonymous hemianopia • Cortical blindness • Ataxia • Dizziness or vertigo • Dysarthria • Diplopia • Dysphagia • Horner’s syndrome • Hemiparesis or hemisensory loss contralateral to the cranial nerves palsy • Cerebellar signs
  • 8. Brain Blood Supply Features: • High oxygen requirement. • Brain 2% of body weight - 15% of cardiac output • 20% of total body oxygen. • Continuous oxygen requirement • Few minutes of ischemia - irreversible injury. • Neurons - Predominantly aerobic. • Sensitive areas: • Adults -Hippocampus, 3,5th & 6th layer of cortex, Purkinje cells. Border zone (watershed areas) • Brain stem nuclei in infants.
  • 11. Frontal[f*ck–motor]Lobe Functions: • High level cognitive functions. i.e reasoning, abstraction, concentration • Storage of information – memory • Control of voluntary eye movement • Motor control of speech in the dominant hemisphere. • Motor Cortex – Motor control of the contralateral side of the body • Urinary continence • Emotion and personality
  • 12. Parietal[p-sx–sensory]Lobe Functions: • Sensory cortex – sensory input is interpreted to define size, weight, texture and consistency (contralateral) • Sensation is localised, and modalities of touch, pressure and position are identified. • Awareness of the parts of body • Non-dominant – processes visuospatial information and • controls spatial orientation • Dominant is involved in ideomotor praxis (ability to perform learned motor tasks
  • 13. Temporal[t-telinga]Lobe Functions: • Primary auditory receptive areas • In dominant ability to comprehend speech (wernicke’s) – reception • Interpretive area – area at the junction of the temporal, parietal and occipital lobes. • Plays an important role in visual, auditory and olfactory perception • Important role in learning; memory and emotional affect.
  • 14. Occipital[O-optic]Lobe Functions: • Primary visual cortex • Visual association areas • Visual perception • Some visual reflexes (i.e. visual fixation) • Involuntary smooth eye movement
  • 15. Diencephalon Functions: • Brain Stem: • Midbrain, Pons & Medulla • 10 of the 12 ranial nerves arise from the brainstem (ipsilateral signs) • Cortical pathway decussation contralateral signs. • Some major functions: eye movement, swallowing, breathing, blood pressure, heat beat, consciousness • Cerebellum: • movement – Balance & coordination
  • 16. Motor & Sensory Cortex:
  • 19. Stroke Types: • Clinical • Transient Ischemic Attack –TIA resolve <24h • Evolving stroke – increasing >24h. – Thromb. • Recurrent / multiple stroke – sec. factors. • Completed stroke – no change… embolic. • Pathological • Focal / Global • Ischemic & hemorrhagic (chronic/acute) • Venous infarcts. (young, infections)
  • 20. Common Types and Incidence: • Infarction: Incidence 80% - mortality 20% • 50% - Thrombotic – atherosclerosis • Large-vessel 30% (carotid, middle cerebral) • Small vessel 20% (lacunar stroke) • 30% Embolic (heart dis / atherosclerosis) • Young, rapid, extensive. • Venous thromboembolism (rare) • Hemorrhage: Incidence 20% - mortality 80% • Berry aneurysm, Microaneurysm, Atheroma. • Intracerebral or subarachnoid.
  • 21. Stroke location and incidence: Cause % Clinical presentation 30day mort(%) Pathogenesis Cerebral infarction 85 Slowly / sudden evolving signs and symptoms 15-45 Cerebral hypoperfusion Embolism Thrombosis Intracerebral hemaemorrha ge 10 Sudden onset of stroke with raised intracranial pressure 80 Rupture of micro- aneurysm or arteriole Subarachnoid haemorrhage 5 Sudden headache with meningism 45 Rupture of saccular aneurysm on circle of Willis
  • 22. Clinical Categories: • Global Ischemia. • Hypoxemic encephalopathy • Hypotension, hypoxemia, anemia. • Focal Ischemia. • Obstruction to blood supply to focal area. • Thrombosis, embolism or hemorrhage.
  • 24. Global Ischemia: • Etiology: • Impaired blood supply - Lung & Heart disorders. • Impaired O2 carrying – Anemia/Blood dis. • Impaired O2 utilization – Cyanide poisoning. • Morphology: • 3rd, 5th and 6th layers of the cortex, hippocampus and in the Purkinje cells in the cerebellum • Laminar necrosis, Hippocampus, Purkinje cells. • Border zone infarcts – “Watershed” • Sickle shaped band of necrosis on cortex. • Clinical Features: • Mild transient confusion state to • Severe irreversible brain death. Flat EEG, Vegetative state. Coma.
  • 25. Morphology in Global Ischemia 1. Watershed zone (Acute - ACA-MCA) 2. Laminar necrosis - (chronic- short penetrating arteries) 3. Sommer sector of hippocampus. 4. Purkinje cells of cerebellum.
  • 27. Lamellar necrosis in global ischemia. Chronic
  • 28. Local infarction: Cell death ~ 6min central infarct area or umbra, surrounded by a penumbra of ischemic tissue that may recover
  • 29. Infarct Pathogenesis: • Reduced blood supply – hypoxia/anoxia. • Altered metabolism  Na/K pump block. • Glutamate receptor act.  calcium influx. • ischemic injury – Red neuron, vacuolation. • cell death, karyorrhexis. • Inflammation – edema. • Macrophages - > 5d. • Liquifaction cavity – >1wk • Glial proliferation – >1wk. (astrocytes) Hours 1-day 3-day 1 wk. >4wk
  • 30. Infarct Stages: • Immediate – <24 hours • No Change gross, micro  Na/K loss, Ca+ influx. • Acute stage – < 1week • Oedema, loss of grey/white matter border. • Inflammation, Red neurons, necrosis, neutrophils • Intermediate stage – 1- 4 weeks. • Clear demarcation, soft friable tissue, cysts • Macrophages, liquifactive necrosis • Late stage – > 4 weeks. • Removal of tissue by macrophages • Fluid filled cysts with dark grey margin (gliosis) • Gliosis – proliferation of glia at periphery.
  • 32. Brain Stem Stroke:CommonPattern • Pure Motor - Weakness of face and limbs on one side of the body without abnormalities of higher brain function, sensation, or vision (MCA/ACA) • Pure Sensory - Decreased sensation of face and limbs on one side of the body without abnormalities of higher brain function, motor function, or vision (PCA).
  • 33.
  • 34. MCA [mostcommon]Features: • Paralysis of the contralateral face, arm and leg • Sensory impairment over the contralateral face, arm & leg • Homonymous hemi or quadrantonopia • Paralysis of gaze to the opposite side • Aphasia (dominant) and dysarthria [broca/wernicke] • Penetrating - contralateral hemiplegia/paresis, slurred speech. • Impaired spatial perception
  • 36.
  • 37. MCA stroke. Wikipedia: GNU Free Documentation license
  • 38. MCA stroke. Wikipedia: GNU Free Documentation license
  • 39. ACA stroke. • Paralysis of contralateral foot and leg • Sensory loss over toes, foot and leg • Impairment of gait and stance • Abulia (slowness and prolonged delays to perform acts) • Flat affect, lack of spontaneity, slowness, distractibility • Cognitive impairment, such as perseveration and amnesia • Urinary incontinence Wikipedia: GNU Free Documentation license
  • 40.
  • 41. PCA stroke. Peripheral (cortical) • Homonymous hemianopia • Memory deficits • Perseveration • Several visual deficits (cortical blindness, lack of depth perception, hallucinations) Central (penetrating) • Thalamus - contralateral sensory loss, spontaneous pain, mild hemi • Cerebral peduncle - CN III palsy with contralateral hemiplegia • Brain stem - CN palsies, nystagmus, pupillary abnormalities Wikipedia: GNU Free Documentation license
  • 42. Posterior Cerebral Artery • Visual disturbances • contralateral homonymous hemianopsia • (central vision is often spared) • L. Hemi: lesions alexia • (with or without agraphia) • Bilateral lesions: cortical blindness • patients unaware they cannot see • (Anton's syndrome) • Memory impairment if temporal lobe is affected ~ • Proximal occlusion • contralateral hemisensory loss, • spontaneous pain and dysesthesia if thalamus affected • (thalamic pain syndrome) • contralateral severe proximal chorea • (hemiballism) ~
  • 43.
  • 44.
  • 45. Haemorrhagic - Arterial embolus Embolic stroke: sudden, pin point hemorrhages over a triangular area.
  • 46. Infarct with Punctate hemorrhage
  • 49. Hypertensive CVD • Intraerebral/Subarachnoid Hemorrhage • Microaneurysm hemorrhages – Basal ganglia. Putamen(60%), thalamus, ventricles. • Berry aneurysm hemorrhages – subarachnoid. • Chronic Hypertension: (dementia) • Slit hemorrhages. Microhemorrhages heal as slit with pigment. • Lacunar infarcts: Brain stem - pale infarcts. A.sclerosis • Hypertensive encephalopathy-Malignant. • Headache, confusion, vomiting – Raised ICP.
  • 50.
  • 56.
  • 57.
  • 60. Left(Dominant)HemisphereStroke:Clinical • Aphasia • Right hemiparesis • Right-sided sensory loss • Right visual field defect • Poor right conjugate gaze • Dysarthria • Difficulty reading, writing, or calculating Diagnosis: Recent cerebral infarction in left MCA distribution. Left cerebral hemisphere shows swelling with compression of the lateral ventricle mainly in the frontal area, due to recent infarct in the Middle Cerebral Artery (MCA) distribution. The brain in the MCA area shows discoloration of the cortex and also blurring between the cortex and white matter.
  • 61. Right(Non-dominant)-HemisphereStroke: • Defect of left visual field • Extinction of left-sided stimuli • Left hemiparesis • Left-sided sensory loss • Left visual field defect • Poor left conjugate gaze • Dysarthria • Spatial disorientation
  • 62. CNS AV Malformations: • Many types: • AV Malformation * • Cavernous angioma • Telangiectasia • Venous angioma • Cause of Seizure disorders & hemorrhage. • Most common congenital vascular malformation. • Typically located in the outer cerebral cortex underlying white matter.
  • 63.
  • 64. Pathological Review: • Stroke: Ischemic / Thrombotic / Hemorrhagic • Acute neurological deficit - Clinical • Cerebro Vascular Accident – Pathology. • Etiology: Thrombosis, Embolism, Hemorrhage. • Risk factors: AS, Hypertension, Smoking. • Global – Systemic Hypoxia – Watershed & lamellar infarct • Focal – Basal ganglia, Putamen, Int. capsule (MCA) • Pathogenesis: Infarction  Liquifaction necrosis  Cyst formation with peripheral gliosis. (loss of neural function) • Hypertension & CVA: • Atherosclerosis - Thrombosis • Haemorrhage (Intra/subarachnoid), • chronic benign: Lacunar infarcts & slit hemorrhages. • Hypertensive Encephalopathy,
  • 65. Stroke – Risk Factors • Modifiable • Hypertension • Tobacco use • Excess Alcohol • Hx of TIA’s • Heart Disease • Diabetes Mellitus • Hypercoagulopathy • Pregnancy, cancer, etc. • Sickle Cell and increased RBC • Hx of carotid Bruit • Unmodifiable • Age • Gender • Race • Previous CVA • Heredity
  • 66.
  • 67. Stroke – Signs and Symptoms • Ischemic • Carotid Circulation • Unilateral paralysis (opposite side) • Numbness (opposite side) • Language disturbance • Aphasia – difficult comprehension, nonsense, difficult reading/writing • Dysarthria – slurred speech, abnormal pronunciation. • Visual disturbance (opposite side) • Monocular blindness (same side)
  • 68. Stroke – Signs and Symptoms • Ischemic • Vertebrobasilar Circulation • Vertigo • Visual disturbance • Both eyes simultaneously • Diplopia • Ocular palsy – inability to move to one side • Dysconjugate gaze – asynchronous movement • Paralysis • Numbness • Dysarthria • Ataxia
  • 69. Stroke – Signs and Symptoms • Hemorrhagic • Subarachnoid hemorrhage • Sudden severe HA • Transient LOC • Nausea/Vomiting • Neck pain • Intolerance of noise/light • AMS • Intracerebral hemorrhage • Focal sx w/ LOC, N/V
  • 70. History • Detailed history from relative or friend or patient if he is able to speak. • Rapidity of onset – sudden onset of a focal neurological deficit. • Time course of symptoms – maximum deficit over seconds or minutes before starting to improve. • Headache, coma at onset and vomiting at onset are more common in haemorrhage but also occur with infarction. • Sudden onset of severe generalised headache associated with neck stiffness – subarachnoid haemorrhage. • Specific record should be made about the presence of vascular risk factors.
  • 71. Examination General Examination • BP – should be taken in both arms. • Stroke may cause an acute rise in BP and therefore hypertension should not be diagnosed in the first few days after a stroke unless left ventricular hypertrophy of fundal changes are present. • Pulse – for arrhythmias particularly atrial fibrillation. • Peripheral pulses. • Auscultation for a carotid bruit. • Heart – for valvular heart disease especially mitral stenosis. • Neck stiffness – subarachnoid haemorrhage or meningitis. • Identify the anatomical localization of the lesion and record the degree of disability.
  • 73.
  • 74.
  • 75.
  • 80. Subarachnoid haemorrhage • Initial headache or coma – sudden rise in intracranial pressure. • Focal symptoms if aneurysm ruptures into underlying brain. • Cerebral vasospasm causes delayed cerebral infarction 4-14 days after onset in 30% of patients. • Recurrent haemorrhage and hydrocephalus are other complications.
  • 81. Intracerebral Haemorrhage • Sudden rupture of microaneurysms caused by hypertensive vascular disease. • Characteristically occurs in the basal ganglia, pons and cerebellum. • Elderly patients – cerebral amyloid angiopathy, a degenerative disorder affecting the walls of the artey – subcortical haematomas. • Cryptic av malformations are suspect especially in younger patients < 40 yrs and when the haemotoma is Lobar (frontal, temporal, parieto-occipital).
  • 82. Investigations Confirm the diagnosis CT Scan  To establish the site, size and pathological diagnosis by showing infarction or haemorrhage.  To exclude other conditions that may mimic stroke like subdural haematoma, intracranial tumour.
  • 83. (a) There is doubt about the diagnosis. (b) Symptoms progress or fluctuate. (c) Conscious level is depressed or patient is in coma. (d) If thrombolytic therapy or anticoagulant treatment is considered. (e) Neck stiffness is present. (f) Has severe headache. (g) Deteriorates unexpectedly. (h) Haemorrhage can be seen within a few minutes as an area of increased attenuation. (i) Infarction as a low density lesion which conforms to a vascular territory usually wedge shaped. (j) It is not immediately visible on CT but in most patients becomes apparent in 4-7 days. Y CT Scan?
  • 84.
  • 85.
  • 86.
  • 87. MRI Scan • Posterior circulation strokes are more readily identified than by CT.
  • 88. General Investigations • identify conditions which may predispose towards premature cerebrovasculardisease. • Full blood count – polycythemia, thrombocytopoenia. • Blood glucose – diabetes mellitus. • Serum lipids – hypercholesterolemia. • Blood cultures – SBE.
  • 89. General Investigations II • HIV screen – AIDS. • Syphilis serology – VDRL. • Clotting Screen. • Thrombophilia Screen – Protein C, Protein S, Antithrombin III. • Anticardolipin antibodies – SLE. • Lumbar Puncture – subarachnoid haemorrhage.
  • 90. DD Stroke • Differential Diagnosis of Stroke • Head/Cervical trauma • Meningitis/encephalitis • Hypertensive encephalopathy • Intracranial mass • Tumor • Sub/epi dural hematoma • Todd’s paralysis • Migraine w/ neuro sx • Metabolic • Hyper/hypo glycemia • Post arrest ischemia • Drug OD
  • 91. Differential Diagnosis • Space occupying lesion • 5% of people with stroke like symptoms have a subdural haematoma, tumour or cerebral abscess. • Distinction is readily made on CT or MRI. • If there is any doubt repeat the scan after 6 weeks.
  • 92. Differential Diagnosis II • Multiple sclerosis • May present with hemiparesis, sensory impairment or brainstem symptoms that mimic stroke. • Symptoms occur gradually over a few days. • Hypoglycaemia • May cause hemiparesis. • Migraine
  • 93. Complications I • Cerebral oedema •Should be suspected in a patient with a large infarct or haemorrhage experiences a lucid interval of 24-48 hours and then shows a decline in consciousness.
  • 94. Complication II • Haemorrhagic transformation • Can occur as a result of thrombolysis. • Pneumonia • In patients with swallowing difficulties as a result of aspiration. • Pressure sores • Develop rapidly and may be exacerbated because of incontinence. • Oedema of Weak Limbs • Is common and has a partially autonomic basis.
  • 95. Complication III • Anxiety and Depression • Common reactions to stroke but depression may have an organic basis related to damage of the frontal and limbic systems. • Emotional Lability • Precipitated by minor emotional stimuli.
  • 96. Management I • Has the following aims • Confirmation of the diagnosis, anatomical site of the lesion, pathology and aetiology. • Acute care. • Rehabilitation of persistent disability and handicap. • Prevention of recurrence.
  • 97. Management II Acute Care Treatment aims • Prevent progression of present event. • Prevent immediate complication. • Prevent the development of subsequent events. • To rehabilitate the patient.
  • 98. Management III General Measures • Around the edge of the infarct, ischaemic tissue is at risk, but is potentially recoverable. • This must be protected by ensuring a good supply of glucose and oxygen. • Maintain hydration, oxygenation and blood pressure.
  • 99. Specific Measures MedicalTreatment Anticoagulation • Patient with high risk of developing deep vein thrombosis. • Thromboembolic stroke - started as soon as possible, except in large infarcts where it may be wise to delay anticoagulation for 2 weeks. • Stroke in a patient with myocardial infarct - due to mural thrombus. • Stroke in evolution. Exclude a haemorrhage by doing a CT scan first.
  • 100. Medical Treatment II Antiplatelet Agents • Especially in TIA. Thrombolysis • I/V thrombolysis espcially recombinant tissue plasminogen activator rTPA to be given only within the first 3 hours after onset to those patients who have not developed CT abnormalities especially in patients with basilar artery occlusion. • Risk – haemorrhage.
  • 101. Hypertension • Treated cautiously in acute stroke. • A reduction in blood pressure may lower cerebral blood flow in the regions surrounding the infarct below a critical level at which further ischaemic brain damage will occur. • Mild to moderate elevations in BP – no treatment unless they are maintained for several days after admission.
  • 102. • If stroke associated with hypertensive encephalopathy or if diastolic BP is persistently above 120 mm Hg. • The BP should be lowered cautiously using oral agents. • Sudden precipitious fall in BP should be avoided. Hypertension II
  • 103. Medical Treatment CalciumAntagonist • Nimodipine prevents ischaemic brain damage and reduces the number of patients remaining disabled after subarachnoid haemorrhage. • Prescribed as soon as diagnosis is made (within 12 hours).
  • 104. Neurosurgery • Should be considered in subaracnoid and intracerebral haemorrhage. • Evacuation of cerebellar haematoma. • Evacuation of supratentorial haematomas should only be considered in younger patients with superficial cortical haematomas causing mass effect with a deteriorating level of conciousness.
  • 105. Nursing Care and Rehabilitation • Physiotherapy, proper positioning and early mobilization – prevent pressure sores. • Support stockings – prevent deep vein thrombosis and pulmonary embolism. • Swallowing difficulties. • Lead to silent aspiration – aspiration pneuomonia. •Nasogastric tube feeding. • Percutanious endoscopic gastrostomy.
  • 106. Rehabilitation • Physiotherapy, occupational therapy, speech therapy and psychology input – multidisciplinary stroke rehabilitation team. • Home visit by occupational therapist to plan adaptations to home before discharge.
  • 107. Secondary Preventation • Control hypertension and diabetes mellitus. • Correct lipid abnormality. • Stop cigarette smoking. • Stop thrombogenic drugs e.g. oral contraceptives.
  • 108. • Give platlet antiaggregation drugs to reduce the rate of reinfarction. • Low dose aspirin (75 mg – 150 mg), if patient allergic or has gastrointestinal side effects give ticlopidine. Regular blood tests because of a small risk of neutropoenia. • Remove or treat embolic source (long term anticoagulation in atrial fibrillation). • Treat inflammatory or vascular inflammatory diseases. Secondary Preventation II
  • 109. • Carotid Endarterectomy – preventing stroke in symptomatic patients with recent TIA and stroke and severe stenosis of the internal carotid artery (at least 70%). Secondary Preventation III
  • 110. Chain of Survival Stroke • Stroke Chain of Survival • Detection • Early sx recognition • Dispatch • Prompt EMS response • Delivery • Transport, approp, prehospital care, prearrival notification • Door • ER Triage • Data • ER evaluation incl, CT, etc. • Decision • Appropriate therapies • Drug/Therapy
  • 111. Detect & Dispatch • Detection: Early Recognition • Public education of Stroke sx • Early access to medical care • Dispatch: Early EMS and PDI’s • Caller triage • EMD recognition of Stroke sx
  • 112. How to detect? • Delivery: • Pre-hospital Transport and Management How we scale the pre- hospital management of the patient?
  • 114. CINCINNATI STROKE SCALE • Identifies patients with strokes. • It evaluates three major physical findings. •Facial droop •Motor arm weakness •Speech abnormalities
  • 115. CSS - Facial Droop • Have the patient show their teeth or smile. • Normal – both sides of the face move equally well • Abnormal – one side of the face does not move as well as the other side
  • 116.
  • 117.
  • 118. Arm Drift • Have the patient close his/her eyes and hold both arms out. • Normal – both arms move the same way, or both arms do not move at all. • Abnormal – one arm does not move or one arm drifts down compared to the other arm. Other findings such as pronator grip, may be helpful
  • 119.
  • 120.
  • 121. Speech • Have the patient say “You can’t teach an old dog new tricks.” – “Perlambagaan Malaysia” • Normal – patient uses correct words with no slurring. • Abnormal – patient slurs words, uses inappropriate words, or is unable to speak
  • 122. “You can’t teach an old dog new tricks.”
  • 123.
  • 124. Cincinnati Prehospital Stroke Scale • Patients with 1 of these 3 findings -as a new event - have a 72% probability of an ischemic stroke. • If all 3 findings are present the probability of an acute stroke is more than 85% • Immediately contact medical control and the destination ED and provide prearrival notification.
  • 125. Stroke – Management In Review: Prehospital Critical Actions • Assess and support cardiorespiratory function • Assess and support blood glucose • Assess and support oxygenation and ventilation • Assess neurologic function • Determine precise time of symptom onset • Determine essential medical information • Provide rapid emergent transport to ED • Notify ED that a possible stroke patient is en route
  • 126. Stroke - Management • Door: ER Triage • Stroke evaluation targets for stroke patients who are thrombolytic candidates Door-to–doctor first sees patient …….………… 10 min Door-to–CT completed …….………………….. 25 min Door-to–CT read ...…………..………………… 45 min Door-to–fibrinolytic therapy starts …………….. 60 min Neurologic expertise available* …..…………… 15 min Neurosurgical expertise available* …………… 2 hours Admitted to monitored bed ..……...…………… 3 hours *By phone or in person
  • 127. Stroke - Management • Data: ER Evaluation and Management • Assessment Goal: in first 10 minutes • Assess ABCs, vital signs • Provide oxygen by nasal cannula • Obtain IV access; obtain blood samples (CBC, ’lytes, coagulation studies) • Obtain 12-lead ECG, check rhythm, place on monitor • Check blood sugar; treat if indicated • Alert Stroke Team: neurologist, radiologist, CT technician • Perform general neurologic screening assessment
  • 128. Stroke - Management Assessment Goal: in first 25 minutes • Review patient history • Establish symptom onset (<6 hours required for fibrinolytics) • Perform physical examination • Perform neurologic exam • Determine level of consciousness (Glasgow Coma Scale) • Determine level of stroke severity (NIHSS or Hunt and Hess Scale) • Order urgent non-contrast CT scan/angiogram if non-hemorrhage (door-to–CT scan performed: goal <25 min from arrival) • Read CT scan (door-to–CT read: goal <45 min from arrival) • Perform lateral cervical spine x-ray (if patient comatose/trauma history)
  • 129. Stroke - Management • ER Diagnostic Studies • CT scan – done w/in 25 mins, read w/in 45 mins • r/o hemorrhage • Often normal early in ischemic stroke • Lumbar puncture • EKG • Changes may be caused by or cause of stroke • MRA (Magnetic Resonance Angiography) • Cerebral Angiography
  • 130. Hypodense area: • Ischemic area with edema, swelling • Indicates >3 hours old • No fibrinolytics!
  • 131. (White areas indicate hyperdensity = blood) Large left frontal intracerebral hemorrhage. Intraventricular bleeding is also present No fibrinolytics!
  • 132. Acute subarachnoid hemorrhage Diffuse areas of white (hyperdense) images Blood visible in ventricles and multiple areas on surface of brain
  • 133. Stroke - Management • Decision: Specific Therapies • General Care • ABC’s, O2 • IV w/ BSS • Treat hypotension • Avoid over-hydration • Monitor input/output • Normalize BGL • Manage Elevated BP?
  • 134. Stroke - Management • Indications for Antihypertensive therapy In general: • Consider: absolute level of BP? • If BP: >185/>110 mm Hg = fibrinolytic therapy contraindicated • Consider: other than BP, is patient candidate for fibrinolytics? • If patient is candidate for fibrinolytics: treat initial BP >185/>110 mm Hg • Consider: response to initial efforts to lower BP in ED? • If treatment brings BP down to <185/110 mm Hg: give fibrinolytics • Consider: ischemic vs hemorrhagic stroke? • Treat BP in the 180-230/110-140 mm Hg range the same • The obvious: no fibrinolytics for hemorrhagic stroke
  • 135. Stroke - Management • Decision: Specific Therapies (cont.) • Management of Seizures • Benzodiazepines • Long-acting anticonvulsants • Management of Increased ICP • Maintain PaCO2 30mm Hg • Mannitol/Diuretics • Barbiturates • Neurosurgical decompression
  • 136. Stroke - Management • Drugs: Thrombolytic Therapy • Fibrinolytic Therapy Checklist Ischemic Stroke Candidates for Neurointerventional Therapy  Age 18 years or older  Acute signs and symptoms of CVA <6 hours onset.  No contraindications.
  • 137. Stroke - Management Contraindications for Interventional Therapy Absolute  Evidence of intracranial hemorrhage on non-contrast head CT  Patient with early infarct signs on CT scan. Relative  Recent (w/in 2 mo’s) cranial or spinal surgery, trauma, or injury  Known bleeding disorder and/or risk of bleeding including: - Current anticoagulant therapy, prothrombin time >15 sec. - Heparin within 48 hrs of admission, PTT elevated - Platelet count <100,000/mm  Active internal bleeding w/in the previous 10 days  Known or suspected pregnancy  History of stroke w/in past 6 weeks
  • 138. Stroke - Management Contraindications for Interventional Therapy (cont.) Relative  Patient comatose  >85 years old  Diabetic hemorrhagic retinopathy or other opthalmic hemorrhagic disorder  Advanced liver or kidney disease  Other pathology with a propensity for bleeding  Infectiouse endocarditis  Severe EKG disturbance, uncontrolled angina or acute MI
  • 139. Stroke - Management • Thrombolytic Agents • TPA • NINDS trial • Streptokinase • VEGGIE trial • Anticoagulant Therapy • Heparin • ASA/Warfarin/Ticlodipine
  • 140. Stroke - Management • Management of Hemorrhagic Stroke • Subarachnoid • Neurosurgical intervention • Nimodipine • Intracerebral • Management of ICP • Neurosurgical decompression • Cerebellar • Surgical evacuation • Often associated with good outcome • Lobar • Surgical evacuation
  • 141. 14 1
  • 143. If they come to you with stroke, what investigation what you like to do?
  • 144.
  • 147. QUESTION 1 On the history of a patient with suspected or known stroke, what should you ask about?
  • 148. ANSWER 1  Ask about the presenting symptoms of stroke initially, for example 1. Unilateral weakness or clumsiness 2. Difficulty understanding or expressing spoken language 3. Altered sensation unilaterally 4. Partial or complete loss of vision in one eye
  • 149. ANSWER 1  Questions to focus on the causes of stroke 1. Atherosclerosis, e.g. enquire about vascular risk factors (smoking, DM, hyperlipids, PVDs, etc…) 2. History of heart disease, e.g. recent myocardial infarction, history of AF requiring anticoagulation, palpitations 3. History of hypertension (lacunar infarcts due to arteriosclerosis of small penetrating arteries of the brain) 4. Migraine 5. Manipulation of neck (precipitating cause for dissection of carotid artery or vertebral artery) 6. Any recent cessation of anticoagulation 7. Family history of stroke 8. History of connective tissue disease (e.g. SLE, vasculitis, etc…)
  • 150. ANSWER 1 1. History of connective tissue disease (e.g. SLE, vasculitis, etc…)  Medication history, esp. those that increase risk of stroke include oral contraceptives, some antihypertensives  Ask about alcohol consumption and recent falls (may have caused an intracranial haemorrhage)  Enquire about premorbid as well as the current level of independence and mobility  If patient is incapacitated, ask about social support available at home  Don’t forget to screen for depression
  • 151. QUESTION 2 On examination, what features should you be looking for?
  • 152. ANSWER 2 A complete examination of the neurological and cardiovascular systems is essential  Check the fundi for evidence of emboli, hypertensive changes, diabetic changes and ischaemic neuropathy  Test the visual fields for homonymous hemianopia  Listen for bruit over the carotids and orbits (commonly heard in the side opposite the carotid occlusion, due to increased contralateral flow)  Decide whether patient is in AF  Assess BP and test for postural drop  Listen for murmurs (e.g. AS, infective endocarditis, rheumatic heart disease or prosthetic valve)  Note presence of electronic pace maker and assess whether it’s working  Perform peripheral vascular examination  Look for complications, e.g. pressure sores, limb contractures and disuse atrophy of the paralysed limbs  See if patient has a percutaneous gastrostomy (PEG) feeding tube inserted and, if present, inspect for cellulitis or pus around the insertion site
  • 153. QUESTION 3 What clinical features are suggestive of a carotid arterial stroke?
  • 154. ANSWER 3  Transient aphasia  Ipsilateral amaurosis fugax  Contralateral hemiplegia/monoplegia  Contralateral hemi or monoparesis  Carotid bruits (a/w >50% stenosis)
  • 155. QUESTION 4 What clinical features are suggestive of vertebrobasilar insufficiency?
  • 156. ANSWER 4 All the Ds  Dizziness (vertigo and ataxia)  Diplopia +/- blindness  Dysphagia  Dysarthria  ‘Demi-anaesthesia’ – ipsilateral face, contralateral limb  Quadraparesis – indicates basilar artery involvement
  • 157. QUESTION 5 What clinical features are suggestive of a middle cerebral artery stroke?
  • 158. ANSWER 5  Contralateral hemiplegia, hemiparesis, homonymous hemianopia  UMN face, arm > leg, eyes deviated to side of lesion  Aphasia (if dominant lobe) 1. Expressive dysphasia, arm + leg involvement – anterior MCA 2. Receptive dysphasia, visual fields defects – posterior MCA  If non-dominant – speech + comprehension intact
  • 159. QUESTION 6 What clinical features are suggestive of a posterior cerebral artery stroke?
  • 160. ANSWER 6  Homonymous hemianopia (complete)  Cortical blindness  Hemisensory loss  Ipsilateral III nerve palsy
  • 161. QUESTION 7 What clinical features are suggestive of an anterior cerebral artery stroke?
  • 162. ANSWER 7  Contralateral monoplegia (UMN)  Confusion, behaviour disturbance  Grasp reflex  Urinary incontinence
  • 163. QUESTION 8 What clinical features are suggestive of a lacunar syndrome?
  • 164. ANSWER 8  Clumsy hand/dysarthria syndrome – lesion in mid pons  Leg paresis + ataxia – pons or internal capsule  Pure sensory stroke – usually thalamic  Pure motor stroke (arm>leg) - pons or internal capsule
  • 165. QUESTION 9 What investigations would you perform in a stroke patient? – MAINLY TO DIAGNOSE ANY STROKE….
  • 166. ANSWER 9 CT or MRI of the head, looking for ischaemic infarcts, haemorrhage, or mass lesions
  • 167. QUESTION 10 What investigations would you perform in a stroke patient? MAINLY TO FIND THE CAUSE OF STROKE
  • 168. ANSWER 10  Doppler scan of the carotid arteries – if duplex scan suggest significant carotid stenosis, esp. in patients <75, ask for results of carotid angiography, carotid digital substraction angiography or MR angiography  If patient is in AF, ask for results of transoesophageal echo, looking for thrombus or spontaneous atheromatous plaques in ascending aorta and arch of aorta that may have contributed to stroke  ECG for AMI and AF  FBE, looking at Hb (to exclude polycythemia) and platelet count (rarely, essential thrombocytopenia can contribute to stroke)  ESR (to exclude an inflammatory arteritic/vasculitic process  CXR (for cardiomegaly/neoplasm)  Urea, creatinine and electrolytes
  • 169. QUESTION 11 What investigations would you perform in a stroke patient? MAINLY IN YOUNGER PATIENT?
  • 170. QUESTION 11  Drug screen, looking for narcotic agents  Vasculitic screen (if there are features of vasculitis)  Blood cultures and cardiac imaging if endocarditis is suspected  Cardiac event monitor looking for paroxysmal AF and  Thrombophilic screen
  • 171. WHAT 3 SPECIFIC STRATEGIES IN THE MANAGEMENT OF ACUTE STROKE THAT HAVE BEEN PROVEN TO IMPROVE OUTCOMES? The 3 main strategies are  Administration of iv tissue plasminogen activator (tPA) within 3hrs of stroke onset  Giving aspirin (100mg) within first 48hrs of ischaemic stroke  Managing patient in a stroke unit Note that tPA and aspirin are never given before brain imaging to exclude intracranial haemorrhage
  • 172. WHAT ARE THE STRATEGIES FOR SECONDARY PREVENTION OF STROKE? There are now at least 4 early strategies for secondary prevention in improving long term outcomes after TIAs or stoke  Antiplatelet therapy, e.g. aspirin, clopidogrel, aspirin with dipyridamole  Blood pressure lowering  Warfarin (indicated for patients with AF)  Carotid endarterectomy (indicated when stenosis >70%)  Lipids lowering is the fifth strategy to consider
  • 173. A PATIENT JUST HAD A STROKE, WHAT CRITERIA DO YOU LOOK FOR TO SELECT THIS PATIENT FOR REHAB. 1. Patients must be medically stable (i.e. no aspiration pneumonia, AMI, DVT, etc…) 2. They have a functional disability 3. They have the intellectual capability to learn 4. There are defined goals to be achieved 5. There are non or minimal co-morbidities, e.g. recurrent stroke, AMI, COAD, etc… 6. They are not clinically depressed (this can affect motivation to rehabilitate) 7. They didn’t suffer from a dense stroke or ones that causes hemi- neglect (e.g. a right hemisphere stroke)
  • 174. WHAT ARE SOME OF THE COMMON PREDICTORS OF POOR FUNCTIONAL OUTCOME AFTER A STROKE?  A dense stroke, a recurrent stroke, or a stroke resulting in hemi- neglect  Impairment of bladder and bowel function  Depression or cognitive deficits  Delayed acute medical care or delayed rehabilitation  Co-morbidities and poor social support

Notas del editor

  1. Impaired spatial perception more likely after R. Hemi. damagespatial neglect dressing apraxiaconstructional apraxiatopographagnosia ~